final exam practice questions

¡Supera tus tareas y exámenes ahora con Quizwiz!

A client is admitted to the hospital with pernicious anemia. The nurse should prepare to administer which of the following medications? A. Folic acid. B. Vitamin B12. C. Lactulose. D. Magnesium sulfate.

Ans: B Rationale: Pernicious anemia is characterized by vitamin B12 deficiency. Magnesium sulfate, lactulose, and folic acid do not address the pathology of this type of anemia.

A client with vitamin D deficiency is receiving education from the nurse. What would be an appropriate recommendation by the nurse? A. Spend time outdoors at least twice per week. B. Increase intake of leafy green vegetables. C. Promote intake of clear fluids. D. Eat red meat at least once per week.

ANS: A Rationale: Skin exposed to ultraviolet light can convert substances necessary for synthesizing vitamin D (cholecalciferol). It is estimated that most people need 5 to 30 minutes of sun exposure twice a week in order for this synthesis to occur. Increasing intake of water is not related to resolving vitamin D deficiency. Vitamin D is unrelated to meat and vegetable intake.

A young student comes to the school nurse and shows the nurse a mosquito bite. As the nurse expects, the bite is elevated and has serous fluid contained in the dermis. How would the nurse classify this lesion? A. Vesicle B. Macule C. Nodule D. Wheal

ANS: A Rationale: A vesicle is a primary skin lesion that is elevated and has fluid contained in the dermis. Examples of vesicles would be a blister or insect bite. Wheals, macules, and nodules are not characterized by elevation and the presence of serous fluid.

A 55-year-old woman is scheduled to have a chemical face peel. The nurse is aware that the client is likely seeking treatment for which of the following? A. Wrinkles near the lips and eyes B. Removal of acne scars C. Vascular lesions on the cheeks D. Real or perceived misshaping of the eyes

ANS: A Rationale: Chemical face peeling is especially useful for wrinkles at the upper and lower lip, forehead, and periorbital areas. Chemical face peeling does not remove acne scars, remove vascular lesions, or reshape the eyes.

A client has just been diagnosed with psoriasis and frequently has lesions around his right eye. What should the nurse teach the client about topical corticosteroid use on these lesions? A. Cataract development is possible. B. The ointment is likely to cause weeping. C. Corticosteroid use is contraindicated on these lesions. D. The client may develop glaucoma.

ANS: A Rationale: Clients using topical corticosteroid preparations repeatedly on the face and around the eyes should be aware that cataract development is possible. Weeping and glaucoma are less likely. There is no consequent risk of glaucoma.

A client with human immune deficiency virus (HIV) has sought care because of the recent development of new skin lesions. The nurse should interpret these lesions as most likely suggestive of what situation? A. A reduction in the client's CD4 count B. A reduction in the client's viral load C. An adverse effect of antiretroviral therapy D. Virus-induced changes in allergy status

ANS: A Rationale: Cutaneous signs may be the first manifestation of human immune deficiency virus (HIV), appearing in more than 90% of HIV-infected people as immune function deteriorates. These skin signs correlate with low CD4 counts and may become very atypical in immunocompromised people. Viral load increases, not decreases, as the disease progresses. Antiretrovirals are not noted to cause cutaneous changes, and viruses do not change an individual's allergy status.

A nurse is conducting a health interview and is assessing for integumentary conditions that are known to have a genetic component. What assessment question is most appropriate? A. "Does anyone in your family have eczema or psoriasis?" B. "Have any of your family members been diagnosed with malignant melanoma?" C. "Do you have a family history of vitiligo or port-wine stains?" D. "Does any member of your family have a history of keloid scarring?"

ANS: A Rationale: Eczema and psoriasis are known to have a genetic component. This is not true of any of the other listed integumentary disorders.

A nurse in the emergency department (ED) is triaging a 5-year-old who has been brought to the ED by the parents for an outbreak of urticaria. What would be the most appropriate question to ask this client's parents? A. "Has your child eaten any new foods today?" B. "Has your child bathed in the past 24 hours?" C. "Did your child go to a friend's house today?" D. "Was your child digging in the dirt today?"

ANS: A Rationale: Foods can cause skin reactions, especially in children. In most cases, this is a more plausible cause of urticaria than bathing, contact with other children, or soil-borne pathogens.

A client's blistering disorder has resulted in the formation of multiple lesions in the client's mouth. What intervention should be included in the client's plan of care? A. Provide chlorhexidine solution for rinsing the client's mouth. B. Avoid providing regular mouth care until the client's lesions heal. C. Liaise with the primary provider to arrange for parenteral nutrition. D. Encourage the client to gargle with a hypertonic solution after each meal.

ANS: A Rationale: Frequent rinsing of the mouth with chlorhexidine solution is prescribed to rid the mouth of debris and to soothe ulcerated areas. A hypertonic solution would be likely to cause pain and further skin disruption. Meticulous mouth care should be provided and there is no reason to provide nutrition parenterally.

A nurse is caring for a client who has been diagnosed with psoriasis. The nurse is creating an education plan for the client. What information should be included in this plan? A. Lifelong management is likely needed. B. Avoid public places until symptoms subside. C. Wash skin frequently to prevent infection. D. Liberally apply corticosteroids as needed.

ANS: A Rationale: Psoriasis usually requires lifelong management. Psoriasis is not contagious. Many clients need reassurance that the condition is not infectious, not a reflection of poor personal hygiene, and not skin cancer. Excessive frequent washing of skin produces more soreness and scaling. Overuse of topical corticosteroids can result in skin atrophy, striae, and medication resistance.

A client presents at the free clinic with a black, wart-like lesion on his face, stating, "I've done some research, and I'm pretty sure I have malignant melanoma." Subsequent diagnostic testing results in a diagnosis of seborrheic keratosis. The nurse should recognize what significance of this diagnosis? A. The client requires no treatment unless he finds the lesion to be cosmetically unacceptable. B. The client's lesion will be closely observed for 6 months before a plan of treatment is chosen. C. The client has one of the few dermatologic malignancies that respond to chemotherapy. D. The client will likely require wide excision.

ANS: A Rationale: Seborrheic keratoses are benign, wart-like lesions of various sizes and colors, ranging from light tan to black. There is no harm in allowing these growths to remain because there is no medical significance to their presence.

A nurse is preparing to assist a surgeon in a skin grafting procedure. What can a skin graft can be used for? A. Denuded skin after burns. B. Slow healing wounds. C. Uncomplicated wound closure. D. Infected wounds.

ANS: A Rationale: Skin grafts are commonly used to repair surgical defects such as those that result from excision of skin tumors, to cover areas denuded of skin (e.g., burns), and to cover wounds in which insufficient skin is available to permit wound closure. They are also used when primary closure of the wound increases the risk of complications or when primary wound closure would interfere with function. It is not used for uncomplicated wound closure. Skin grafts are not used for infected wounds.

An 80-year-old client is brought to the clinic by one of the client's children. The client asks the nurse why the client has gotten so many "spots" on the skin. What would be an appropriate response by the nurse? A. "As people age, they normally develop uneven pigmentation in their skin." B. "These 'spots' are called 'liver spots' or 'age spots.'" C. "Older skin is more apt to break down and tear, causing sores." D. "These are usually the result of nutritional deficits earlier in life."

ANS: A Rationale: The major changes in the skin of older people include dryness, wrinkling, uneven pigmentation, and various proliferative lesions. Stating the names of these spots and identifying older adults' vulnerability to skin damage do not answer the question. These lesions are not normally a result of nutritional imbalances.

A nurse is providing self-care education to a client who has been receiving treatment for acne vulgaris. What instruction should the nurse provide to the client? A. "Wash your face with water and gentle soap each morning and evening." B. "Before bedtime, clean your face with rubbing alcohol on a cotton pad." C. "Gently burst new pimples before they form a visible 'head'." D. "Set aside some time each day to squeeze blackheads and remove the plug."

ANS: A Rationale: The nurse should inform the client to wash the face and other affected areas with mild soap and water twice each day to remove surface oils and prevent obstruction of the oil glands. Cleansing with rubbing alcohol is not recommended and all forms of manipulation should be avoided.

A nurse educator is teaching a group of nurses about Kaposi sarcoma. What would the educator identify as characteristics of endemic Kaposi sarcoma? Select all that apply. A. Affects people predominantly in the eastern half of Africa B. Affects men more than women C. Does not affect children D. Cannot infiltrate E. Can progress to lymphadenopathic forms

ANS: A, B, E Rationale: Endemic (African) Kaposi sarcoma affects people predominantly in the eastern half of Africa, near the equator. Men are affected more often than women, and children can be affected as well. The disease may resemble classic KS or it may infiltrate and progress to lymphadenopathic forms.

During a routine examination of a client's fingernails, the nurse notes a horizontal depression in each nail plate. When documenting this finding, the nurse should use which term? A. Splinter hemorrhage B. Beau line C. Paronychia D. Clubbing

ANS: B Rationale: A Beau line is a horizontal depression in the nail plate. Occurring alone or in multiples, these depressions result from a temporary disturbance in nail growth. A splinter hemorrhage is a linear red or brown streak in the nail bed. Paronychia refers to an inflammation of the skinfold at the nail margin. Clubbing describes an increased angle between the nail plate and nail base.

A client has received a diagnosis of irritant contact dermatitis. What action should the nurse prioritize in the client's subsequent care? A. Teaching the client to safely and effectively administer immunosuppressants B. Helping the client identify and avoid the offending agent C. Teaching the client how to maintain meticulous skin hygiene D. Helping the client perform wound care in the home environment

ANS: B Rationale: A focus of care for clients with irritant contact dermatitis is identifying and avoiding the offending agent. Immunosuppressants are not used to treat eczema and wound care is not normally required, except in cases of open lesions. Poor hygiene has no correlation with contact dermatitis.

Assessment of a client's leg reveals the presence of a 1.5-cm circular region of necrotic tissue that is deeper than the epidermis. The nurse should document the presence of what type of skin lesion? A. Keloid B. Ulcer C. Fissure D. Erosion

ANS: B Rationale: A pressure ulcer that is stage 2 or greater is one that extends past the epidermal layer and can develop necrotic tissue. Keloids lack necrosis and consist of scar tissue. A fissure is linear, and erosions do not extend to the dermis.

A nurse is caring for a client whose skin cancer will soon be removed by excision. Which of the following actions should the nurse perform? A. Teach the client about early signs of secondary blistering diseases. B. Teach the client about self-care after treatment. C. Assess the client's risk for recurrent malignancy. D. Assess the client for adverse effects of radiotherapy.

ANS: B Rationale: Because many skin cancers are removed by excision, clients are usually treated in outpatient surgical units. The role of the nurse is to teach the client about prevention of skin cancer and about self-care after treatment. Assessing the client's risk for recurrent malignancy is primarily the role of the health care provider. Blistering diseases do not result from cancer or subsequent excision. Excision is not accompanied by radiotherapy.

A nurse practitioner is seeing a 16-year-old client who has come to the dermatology clinic for treatment of acne. The nurse practitioner would know that the treatment may consist of which of the following medications? A. Acyclovir B. Benzoyl peroxide and erythromycin C. Diphenhydramine D. Triamcinolone

ANS: B Rationale: Benzoyl peroxide and erythromycin gel is among the topical treatments available for acne. Acyclovir is used in the treatment of herpes zoster as an oral antiviral agent. Diphenhydramine is an oral antihistamine used in the treatment of pruritus. Intralesional injections of triamcinolone have been utilized in the treatment of psoriasis.

A nurse practitioner working in a dermatology clinic finds an open lesion on a client who is being assessed. What should the nurse do next? A. Obtain a swab for culture. B. Assess the characteristics of the lesion. C. Obtain a swab for pH testing. D. Apply a test dose of broad-spectrum topical antibiotic.

ANS: B Rationale: If acute open wounds or lesions are found on inspection of the skin, a comprehensive assessment should be made and documented. Testing for culture and pH are not necessarily required, and assessment should precede these actions. Antibiotics are not applied on an empirical basis.

A client comes to the dermatology clinic requesting the removal of epidermal nevi on the client's right cheek. The nurse knows that the procedure especially useful in treating such lesions is what? A. Skin graft B. Laser treatment C. Chemical face peeling D. Free flap

ANS: B Rationale: Lasers are useful in treating cutaneous vascular lesions such as epidermal nevi. Skin grafts, chemical face peels, and free flaps would not be used to remove this lesion.

A client is suspected of developing an allergy to an environmental substance and has been given a patch test. During the test, the client develops fine blisters, papules, and severe itching. The nurse knows that this is indicative of what strength reaction? A. Weak positive B. Moderately positive C. Strong positive D. Severely positive

ANS: B Rationale: The development of redness, fine elevations, or itching is considered a weak positive reaction; fine blisters, papules, and severe itching indicate a moderately positive reaction; and blisters, pain, and ulceration indicate a strong positive reaction.

The nurse is caring for a client who developed a pressure injury as a result of decreased mobility. The nurse on the previous shift has provided client teaching about pressure injuries and healing promotion. The nurse determines that the client has understood the teaching by observing the client: A. perform range-of-motion exercises. B. avoid placing body weight on the healing site. C. elevate body parts that are susceptible to edema. D. demonstrate the technique for massaging the wound site.

ANS: B Rationale: The major goals of pressure injury treatment may include relief of pressure, improved mobility, improved sensory perception, improved tissue perfusion, improved nutritional status, minimized friction and shear forces, dry surfaces in contact with skin, and healing of pressure ulcer, if present. The other options do not demonstrate the achievement of the goal of the client teaching.

A school nurse has sent home four children who show evidence of pediculosis capitis. What is an important instruction the nurse should include in the note being sent home to parents? A. The child's scalp should be monitored for 48 to 72 hours before starting treatment. B. Nits may have to be manually removed from the child's hair shafts. C. The disease is self-limiting and symptoms will abate within 1 week. D. Efforts should be made to improve the child's level of hygiene.

ANS: B Rationale: Treatment for head lice should begin promptly and may require manual removal of nits following medicating shampoo. Head lice are not related to a lack of hygiene. Treatment is necessary because the condition will not likely resolve spontaneously within 1 week.

When caring for a client with toxic epidermal necrolysis (TEN), the critical care nurse assesses frequently for high fever, tachycardia, and extreme weakness and fatigue. The nurse is aware that these findings are potential indicators of what condition(s)? Select all that apply. A. Possible malignancy B. Epidermal necrosis C. Neurologic involvement D. Increased metabolic needs E. Possible gastrointestinal mucosal sloughing

ANS: B, D, E Rationale: Assessment for high fever, tachycardia, and extreme weakness and fatigue is essential because these factors indicate the process of epidermal necrosis, increased metabolic needs, and possible gastrointestinal and respiratory mucosal sloughing. These factors are less likely to suggest malignancy or neurologic involvement, as these are not common complications of TEN.

The nurse is describing the role of plasminogen in the clotting cascade. Where in the body is plasminogen present? A. Myocardial muscle tissue B. All body fluids C. Cerebral tissue D. Venous and arterial vessel walls

Ans: B Rationale: Plasminogen, which is present in all body fluids, circulates with fibrinogen. Plasminogen is found in body fluids, not tissue.

A nurse is providing care for a client who has psoriasis. Following the appearance of skin lesions, the nurse should prioritize what assessment? A. Assessment of the client's stool for evidence of intestinal sloughing B. Assessment of the client's apical heart rate for dysrhythmias C. Assessment of the client's joints for pain and decreased range of motion D. Assessment for cognitive changes resulting from neurologic lesions

ANS: C Rationale: Asymmetric rheumatoid factor-negative arthritis of multiple joints occurs in up to 42% of people with psoriasis, most typically after the skin lesions appear. The most typical joints affected include those in the hands or feet, although sometimes larger joints such as the elbow, knees, or hips may be affected. As such, the nurse should assess for this musculoskeletal complication. GI, cardiovascular, and neurologic function are not affected by psoriasis.

A 35-year-old kidney transplant client comes to the clinic exhibiting new skin lesions. The diagnosis is Kaposi sarcoma. The nurse caring for this client recognizes that this is what type of Kaposi sarcoma? A. Classic B. AIDS related C. Iatrogenic D. Endemic

ANS: C Rationale: Iatrogenic/organ transplant--associated Kaposi sarcoma occurs in transplant recipients and people with AIDS. This form of KS is characterized by local skin lesions and disseminated visceral and mucocutaneous diseases. Classic Kaposi sarcoma occurs predominantly in men of Mediterranean or Jewish ancestry between 40 and 70 years of age. Endemic KS affects people predominantly in the eastern half of Africa. AIDS-related KS is seen in people with AIDS.

An older adult resident of a long-term care facility has been experiencing generalized pruritus that has become more severe in recent weeks. What intervention should the nurse add to this resident's plan of care? A. Avoid the application of skin emollients B. Apply antibiotic ointment, as prescribed, following baths C. Avoid using hot water during the client's baths D. Administer acetaminophen four times daily as prescribed

ANS: C Rationale: If baths have been prescribed, the client is reminded to use tepid (not hot) water and to shake off the excess water and blot between intertriginous areas (body folds) with a towel. Skin emollients should be applied to reduce pruritus. Acetaminophen and antibiotics do not reduce pruritus.

The nurse is performing an initial assessment of a client who has a raised, pruritic rash. The client denies taking any prescription medication and denies any allergies. What would be an appropriate question to ask this client at this time? A. "Is anyone in your family allergic to anything?" B. "How long have you had this abrasion?" C. "Do you take any over-the-counter (OTC) drugs or herbal preparations?" D. "What do you do for a living?"

ANS: C Rationale: If suspicious areas are noted, the client should be questioned about nonprescription or herbal preparations that might be in use. Asking about family allergies, the duration of an abrasion, or the client's occupation may not provide relevant information at this time.

A nurse is working with a family whose 5-year-old child has been diagnosed with impetigo. What educational intervention should the nurse include in this family's care? A. Ensuring that the family knows that impetigo is not contagious B. Teaching about the safe and effective use of topical corticosteroids C. Teaching about the importance of maintaining high standards of hygiene D. Ensuring that the family knows how to safely burst the child's vesicles

ANS: C Rationale: Impetigo is associated with unhygienic conditions; educational interventions to address this are appropriate. The disease is contagious, thus vesicles should not be manually burst. Because of the bacterial etiology, corticosteroids are ineffective.

A client is admitted to the intensive care unit with what is thought to be toxic epidermal necrolysis (TEN). When assessing the health history of the client, the nurse would be alert to what precipitating factor? A. Recent heavy ultraviolet exposure B. Substandard hygienic conditions C. Recent administration of new medications D. Recent varicella infection

ANS: C Rationale: In adults, TEN is usually triggered by a reaction to medications. Antibiotics, anticonvulsant agents, butazones, and sulfonamides are the most frequent medications implicated. TEN is unrelated to UV exposure, hygiene, or varicella infection.

The nurse is performing a comprehensive assessment of a client's skin surfaces and intends to assess moisture, temperature, and texture. The nurse should perform this component of assessment in what way? A. By examining the client under a Wood light B. By inspecting the client's skin in direct sunlight C. By palpating the client's skin D. By performing percussion of major skin surfaces

ANS: C Rationale: Inspection and palpation are techniques commonly used in examining the skin. A client would only be examined under a Wood light if there were indications it could be diagnostic. The client is examined in a well-lit room, not in direct sunlight. Percussion is not a technique used in assessing the skin.

A new nurse notes that the client's fingernail surfaces are pitted. The nurse should suspect the presence of what health problem? A. Eczema B. Systemic lupus erythematosus (SLE) C. Psoriasis D. Chronic obstructive pulmonary disease (COPD)

ANS: C Rationale: Pitted surface of the nails is a definite indication of psoriasis. Pitting of the nails does not indicate eczema, SLE, or COPD.

A client has a diagnosis of seborrhea and has been referred to the dermatology clinic, where the nurse contributes to care. When planning this client's care, the nurse should include what nursing diagnosis? A. Risk for deficient fluid volume related to excess sebum synthesis B. Ineffective thermoregulation related to occlusion of sebaceous glands C. Disturbed body image related to excess sebum production D. Ineffective tissue perfusion related to occlusion of sebaceous glands

ANS: C Rationale: Seborrhea causes highly visible manifestations that are likely to have a negative effect on the client's body image. Seborrhea does not normally affect fluid balance, thermoregulation, or tissue perfusion.

A nurse is leading a health promotion workshop that is focusing on cancer prevention. What action is most likely to reduce participants' risks of basal cell carcinoma (BCC)? A. Teaching participants to improve their overall health through nutrition B. Encouraging participants to identify their family history of cancer C. Teaching participants to limit their sun exposure D. Teaching participants to control exposure to environmental and occupational radiation

ANS: C Rationale: Sun exposure is the best known and most common cause of BCC. BCC is not commonly linked to general health debilitation, family history, or radiation exposure. Certainly, here are the questions formatted according to your instructions:

A client presents at the dermatology clinic with suspected herpes simplex. The nurse knows to prepare what diagnostic test for this condition? A. Skin biopsy B. Patch test C. Tzanck smear D. Examination with a Wood light

ANS: C Rationale: The Tzanck smear is a test used to examine cells from blistering skin conditions, such as herpes zoster, varicella, herpes simplex, and all forms of pemphigus. The secretions from a suspected lesion are applied to a glass slide, stained, and examined. This is not accomplished by biopsy, patch test, or Wood light.

A public health nurse is participating in a health promotion campaign that has the goal of improving outcomes related to skin cancer in the community. What action has the greatest potential to achieve this goal? A. Educating participants about the relationship between general health and the risk of skin cancer B. Educating participants about treatment options for skin cancer C. Educating participants about the early signs and symptoms of skin cancer D. Educating participants about the health risks associated with smoking and assisting with smoking cessation

ANS: C Rationale: The best hope of decreasing the incidence of skin cancer lies in educating clients about the early signs. There is a relationship between general health and skin cancer, but teaching individuals to identify the early signs and symptoms is more likely to benefit overall outcomes related to skin cancer. Teaching about treatment options is not likely to have a major effect on outcomes of the disease. Smoking is not among the major risk factors for skin cancer. Certainly, here are the formatted questions:

A client diagnosed with a stasis ulcer has been hospitalized. There is an order to change the dressing and provide wound care. Which activity should the nurse first perform when providing wound care? A. Assess the drainage in the dressing. B. Slowly remove the soiled dressing. C. Perform hand hygiene. D. Don non-latex gloves.

ANS: C Rationale: The nurse and health care provider must adhere to standard precautions and wear gloves when inspecting the skin or changing a dressing. Use of standard precautions and proper disposal of any contaminated dressing is carried out according to Occupational Safety and Health Administration (OSHA) regulations. Hand hygiene must precede other aspects of wound care.

A client with squamous cell carcinoma has been scheduled for treatment of this malignancy. The nurse should anticipate that treatment for this type of cancer will primarily consist of what intervention? A. Chemotherapy B. Radiation therapy C. Surgical excision D. Biopsy of sample tissue

ANS: C Rationale: The primary goal of surgical management of squamous cell carcinoma is to remove the tumor entirely. Radiation therapy is reserved for older clients, because x-ray changes may be seen after 5 to 10 years, and malignant changes in scars may be induced by irradiation 15 to 30 years later. Obtaining a biopsy would not be a goal of treatment; it may be an assessment. Chemotherapy and radiation therapy are generally reserved for clients who are not surgical candidates.

A client requires a full-thickness graft to cover a chronic wound. How is the donor site selected? A. The largest area of the body without hair is selected. B. Any area that is not normally visible can be used. C. An area matching the color and texture of the skin at the surgical site is selected. D. An area matching the sensory capability of the skin at the surgical site is selected.

ANS: C Rationale: The site where the intact skin is harvested is called the donor site. Selection of the donor site is made to match the color and texture of skin at the surgical site and to leave as little scarring as possible.

A client has just been told that he has deep malignant melanoma. The nurse caring for this client should anticipate that the client will undergo what treatment? A. Chemotherapy B. Immunotherapy C. Wide excision D. Radiation therapy

ANS: C Rationale: Wide excision is the primary treatment for malignant melanoma, which removes the entire lesion and determines the level and staging. Chemotherapy may be used after the melanoma is excised. Immunotherapy is experimental and radiation therapy is palliative.

A nurse is preparing to perform the physical assessment of a newly admitted client. During which of the following components of the assessment should the nurse wear gloves? Select all that apply. A. Palpation of the nail beds B. Palpation of the client's upper extremities C. Palpation of a rash on the client's trunk D. Palpation of a lesion on the client's upper back E. Palpation of the client's finger joints

ANS: C, D Rationale: Gloves are worn during skin examination if a rash or lesions are to be palpated. It is not normally necessary to wear gloves to palpate a client's extremities or fingers unless contact with body fluids is reasonably foreseeable.

A client's health assessment has resulted in a diagnosis of alopecia areata. What nursing diagnosis should the nurse most likely associate with this health problem? A. Chronic pain B. Impaired skin integrity C. Impaired tissue integrity D. Disturbed body image

ANS: D Rationale: Alopecia areata causes hair loss in smaller defined areas. As such, it is common for the client to experience a disturbed body image. Hair loss does not cause pain and does not affect skin or tissue integrity.

While performing an initial assessment of a client admitted with appendicitis, the nurse observes an elevated blue-black lesion on the client's ear. The nurse knows that this lesion is consistent with what type of skin cancer? A. Basal cell carcinoma B. Squamous cell carcinoma C. Dermatofibroma D. Malignant melanoma

ANS: D Rationale: A malignant melanoma presents itself as a superficial spreading melanoma which may appear in a combination of colors, with hues of tan, brown, and black mixed with gray, blue-black, or white. The lesion tends to be circular, with irregular outer portions. BCC usually begins as a small, waxy nodule with rolled, translucent, pearly borders; telangiectatic vessels may be present. SCC appears as a rough, thickened, scaly tumor that may be asymptomatic or may involve bleeding. A dermatofibroma presents as a firm, dome-shaped papule or nodule that may be skin colored or pinkish brown.

A nurse is planning the care of a client with herpes zoster. What medication, if given within the first 24 hours of the initial eruption, can arrest herpes zoster? A. Prednisone B. Azathioprine C. Triamcinolone D. Acyclovir

ANS: D Rationale: Acyclovir, if started early, is effective in significantly reducing the pain and halting the progression of the disease. There is evidence that infection is arrested if oral antiviral agents are given within the first 24 hours. Prednisone is an anti-inflammatory agent used in a variety of skin disorders, but not in the treatment of herpes. Azathioprine is an immunosuppressive agent used in the treatment of pemphigus. Triamcinolone is utilized in the treatment of psoriasis.

A nurse is caring for a client with Hodgkin lymphoma at the oncology clinic. The nurse should identify what main goal of care? A. Cure of the disease B. Enhancing quality of life C. Controlling symptoms D. Palliation

Ans: A Rationale: The goal in the treatment of Hodgkin lymphoma is cure. Palliation is thus not normally necessary. Quality of life and symptom control are vital, but the overarching goal is the cure of the disease.

A 65-year-old man presents at the clinic reporting nodules on both legs. The man tells the nurse that his son, who is in medical school, encouraged him to seek prompt care and told him that the nodules are related to the fact that he is Jewish. What health problem should the nurse suspect? A. Stasis ulcers B. Bullous pemphigoid C. Psoriasis D. Classic Kaposi sarcoma

ANS: D Rationale: Classic Kaposi sarcoma occurs predominantly in men of Mediterranean or Jewish ancestry between 40 and 70 years of age. Most clients have nodules or plaques on the lower extremities that rarely metastasize beyond this area. Classic KS is chronic, relatively benign, and rarely fatal. Stasis ulcers do not create nodules. Bullous pemphigoid is characterized by blistering. Psoriasis characteristically presents with silvery plaques.

A nurse is assessing the skin of a client who has been diagnosed with bacterial cellulitis on the dorsal portion of the great toe. When reviewing the client's health history, the nurse should identify what comorbidity as increasing the client's vulnerability to skin infections? A. Chronic obstructive pulmonary disease B. Rheumatoid arthritis C. Gout D. Diabetes

ANS: D Rationale: Clients with diabetes are particularly susceptible to skin infections. COPD, RA, and gout are less commonly associated with integumentary manifestations.

A nurse is assessing a teenage client with acne vulgaris. The client's mother states, "I keep telling him that this is what happens when you eat as many french fries as he does." What aspect of the pathophysiology of acne should inform the nurse's response? A. A sudden change in client's diet may exacerbate, rather than alleviate, the client's symptoms. B. French fries are one of the foods that are known to directly cause acne. C. Elimination of fried foods from the client's diet will likely lead to resolution within several months. D. Diet is thought to play a minimal role in the development of acne.

ANS: D Rationale: Diet is not believed to play a major role in acne therapy. A change in diet is not known to exacerbate symptoms. However, there does appear to be a correlation between foods high in refined sugars and acne; therefore, these foods should be avoided.

A client comes to the clinic reporting a red rash of small, fluid-filled blisters and is suspected of having herpes zoster. What presentation is most consistent with this diagnosis? A. Grouped vesicles occurring on lips and oral mucous membranes B. Grouped vesicles occurring on the genitalia C. Rough, fresh, or gray skin protrusions D. Grouped vesicles in linear patches along a dermatome

ANS: D Rationale: Herpes zoster, or shingles, is an acute inflammation of the dorsal root ganglia, causing localized, vesicular skin lesions following a dermatome. Herpes simplex type 1 is a viral infection affecting the skin and mucous membranes, usually producing cold sores or fever blisters. Herpes simplex type 2 primarily affects the genital area, causing painful clusters of small ulcerations. Warts appear as rough, fresh, or gray skin protrusions.

A nurse is providing care for a client who has developed Kaposi sarcoma secondary to HIV infection. The nurse should be aware that this form of malignancy originates in what part of the body? A. Connective tissue cells in diffuse locations B. Smooth muscle cells of the gastrointestinal and respiratory tract C. Neural tissue of the brain and spinal cord D. Endothelial cells lining small blood vessels

ANS: D Rationale: Kaposi sarcoma (KS) is a malignancy of endothelial cells that line the small blood vessels. It does not originate in connective tissue, smooth muscle cells of the GI and respiratory tract, or in neural tissue.

A client has just undergone surgery for malignant melanoma. Which of the following nursing actions should be prioritized? A. Maintain the client on bedrest for the first 24 hours postoperative. B. Apply distraction techniques to relieve pain. C. Provide soft or liquid diet that is high in protein to assist with healing. D. Anticipate the need for, and administer, appropriate analgesic medications.

ANS: D Rationale: Nursing interventions after surgery for a malignant melanoma center on promoting comfort, because wide excision surgery may be necessary. Anticipating the need for and administering appropriate analgesic medications are important. Distraction techniques may be appropriate for some clients, but these are not a substitute for analgesia. Bed rest and a modified diet are not necessary.

A nurse is caring for a client whose chemical injury has necessitated a skin graft to the client's left hand. Which statement is true regarding skin graft use? A. This use is not a type of reconstruction. B. Skin grafts form their own blood supply. C. They are only transplanted from another donor. D. Skin is transferred from a distant site to the graft site.

ANS: D Rationale: Skin grafting is a technique in which a section of skin is detached from its own blood supply and transferred as free tissue to a distant (recipient) site. Skin grafting can be used to repair almost any type of wound and is the most common form of reconstructive surgery.

An older adult client, who is bedridden, is admitted to the unit because of a pressure injury that can no longer be treated in a community setting. During assessment, the nurse finds that the ulcer extends into the muscle and bone. At what stage should the nurse document this injury? A. I B. II C. III D. IV

ANS: D Rationale: Stage III and IV pressure injuries are characterized by extensive tissue damage. Stage IV is an ulcer that extends to underlying muscle and bone. Stage III is an ulcer that extends into the subcutaneous tissue. With this type of ulcer, necrosis of tissue and infection may develop. Stage I is an area of erythema that does not blanch with pressure. Stage II involves a break in the skin that may drain.

A 30-year-old client has just returned from the operating room after having a "flap" done following a motorcycle accident. The client's spouse asks the nurse about the major complications following this type of surgery. What would be the nurse's best response? A. "The major complication is when the client develops chronic pain." B. "The major complication is when the client loses sensation in the flap." C. "The major complication is when the pedicle tears loose and the flap dies." D. "The major complication is when the blood supply fails and the tissue in the flap dies."

ANS: D Rationale: The major complication of a flap is necrosis of the pedicle or base as a result of failure of the blood supply. This is more likely than tearing of the pedicle and chronic pain and is more serious than loss of sensation.

A nurse is explaining the importance of sunlight on the skin to a client with decreased mobility who rarely leaves the house. The nurse would emphasize that ultraviolet light helps to synthesize what vitamin? A. E B. D C. A D. C

Ans: B Rationale: Skin exposed to ultraviolet light can convert substances necessary for synthesizing vitamin D (cholecalciferol). Vitamin D is essential for preventing rickets, a condition that causes bone deformities and results from a deficiency of vitamin D, calcium, and phosphorus.

A client who has sustained third-degree facial burns and a facial fracture is undergoing reconstructive surgery and implantation of a prosthesis. The nurse has identified a nursing diagnosis of Low Self Esteem related to use of facial prosthetic secondary to reconstructive surgery. Which nursing intervention would be appropriate for this diagnosis? A. Referring the client to a speech therapist B. Gradually adding soft foods to diet C. Administering analgesics as prescribed D. Teaching the client how to use and care for the prosthesis

ANS: D Rationale: The process of facial reconstruction is often slow and tedious. Because a person's facial appearance affects self-esteem so greatly, this type of reconstruction is often a very emotional experience for the client. Reinforcement of the client's successful coping strategies improves self-esteem. If prosthetic devices are used, the client is taught how to use and care for them to gain a sense of greater independence. This is an intervention that relates to Disturbed Body Image in these clients. None of the other listed interventions relate directly to the diagnosis of Disturbed Body Image.

A nurse is educating a client about the role of B lymphocytes. The nurse's description will include which of the following physiologic processes? A. Stem cell differentiation B. Cytokine production C. Phagocytosis D. Antibody production

Ans: D Rationale: B lymphocytes are capable of differentiating into plasma cells. Plasma cells, in turn, produce antibodies. Cytokines are produced by NK cells. Stem cell differentiation greatly precedes B lymphocyte production.

A nurse is aware that the outer layer of the skin consists of dead cells that contain large amounts of keratin. The physiologic functions of keratin include which of the following? Select all that apply. A. Producing antibodies B. Absorbing electrolytes C. Maintaining acid-base balance D. Physically repelling pathogens E. Preventing fluid loss

ANS: D, E Rationale: The dead cells of the epidermis contain large amounts of keratin, an insoluble, fibrous protein that forms the outer barrier of the skin. Keratin has the capacity to repel pathogens and prevent excessive fluid loss from the body. It does not contribute directly to antibody production, acid-base balance, or electrolyte levels.

The nurse is caring for a client who is admitted to the medical unit for the treatment of a venous ulcer in the area of the lateral malleolus that has been unresponsive to treatment. Which finding is the nurse most likely to identify during an assessment of this client's wound? A. Hemorrhage B. Heavy exudate C. Deep wound bed D. Pale-colored wound bed

Ans: B Rationale: Venous ulcerations in the area of the medial or lateral malleolus (gaiter area) are typically large, superficial, and highly exudative. Venous hypertension causes extravasation of blood, which discolors the area of the wound bed. Bleeding is not normally present.

While waiting to see the health care provider, a client shows the nurse skin areas that are flat, nonpalpable, and have had a change of color. The nurse recognizes that the client is demonstrating: A. macules. B. papules. C. vesicles. D. pustules.

Ans: A Rationale: A macule is a flat, nonpalpable skin color change, while a papule is an elevated, solid, palpable mass. A vesicle is a circumscribed, elevated, palpable mass containing serous fluid, while a pustule is a pus-filled vesicle.

An unresponsive client with a light complexion has been brought to the emergency room by EMS. While assessing this client, the nurse notes that the client's face is a cherry-red color. What should the nurse suspect? A. Carbon monoxide poisoning B. Anemia C. Jaundice D. Uremia

Ans: A Rationale: Carbon monoxide poisoning causes a bright cherry red color in the face and upper torso in light-skinned persons. In dark-skinned persons, there will be a cherry-red color to nail beds, lips, and oral mucosa. When anemia occurs in light-skinned persons, the skin has generalized pallor. Anemia in dark-skinned persons manifests as a yellow-brown coloration. Jaundice appears as a yellow coloration of the sclerae. Uremia gives a yellow-orange tinge to the skin.

The nurse in an ambulatory care center is admitting an older adult client who has bright red moles on the skin. What benign changes in the skin of an older adult appear as bright red moles? A. Cherryangiomas B. Solar lentigines C. Seborrheic keratoses D. Xanthelasmas

Ans: A Rationale: Cherry angiomas appear as bright red "moles," while solar lentigines are commonly called "liver spots." Seborrheic keratoses are described as crusty brown "stuck on" patches, while xanthelasmas appear as yellowish, waxy deposits on the upper eyelids.

The outer layer of the epidermis provides the most effective barrier to penetration of the skin by environmental factors. Which of the following is an example of penetration by an environmental factor? A. An insect bite B. Dehydration C. Sunburn D. Excessive perspiration

Ans: A Rationale: The stratum corneum, the outer layer of the epidermis, provides the most effective barrier to both epidermal water loss and penetration of environmental factors, such as chemicals, microbes, insect bites, and other trauma. Dehydration, sunburn, and excessive perspiration are not examples of penetration of an environmental factor.

A client has been diagnosed with a lymphoid stem cell defect. This client has the potential for a problem involving which of the following? A. Plasma cells B. Neutrophils C. Red blood cells D. Platelets

Ans: A Rationale: A defect in a myeloid stem cell can cause problems with erythrocyte, leukocyte, and platelet production. In contrast, a defect in the lymphoid stem cell can cause problems with T or B lymphocytes, plasma cells (a more differentiated form of B lymphocyte), or natural killer (NK) cells.

A nurse at a blood donation clinic has completed the collection of blood from a client. The client reports feeling "light-headed" and appears pale. Which action by the nurse is most appropriate? A. Help the client to sit, with head lowered below knees. B. Administer supplementary oxygen by nasal prongs. C. Obtain a full set of vital signs. D. Inform a healthcare provider or other primary care provider.

Ans: A Rationale: A donor who appears pale or complains of faintness should immediately lie down or sit with the head lowered below the knees. The client should be observed for another 30 minutes. There is no immediate need for a health care provider's care. Supplementary oxygen may be beneficial, but may take too much time to facilitate before a syncopal episode. Repositioning must precede assessment of vital signs.

A client comes to the clinic reporting fatigue and the health interview is suggestive of pica. Laboratory findings reveal a low serum iron level and a low ferritin level. With what would the nurse suspect that the client will be diagnosed? A. Iron deficiency anemia. B. Pernicious anemia. C. Sickle cell disease. D. Hemolytic anemia.

Ans: A Rationale: A low serum iron level, a low ferritin level, and symptoms of pica are associated with iron deficiency anemia. TIBC may also be elevated. None of the other anemias are associated with pica.

A man tells the nurse that their father died of prostate cancer and the client is concerned about their own risk of developing the disease, having heard that prostate cancer has a genetic link. What aspect of the pathophysiology of prostate cancer would underlie the nurse's response? A. A number of studies have identified an association of BRCA-2 mutation with an increased risk of prostate cancer. B. HNPCC is a mutation of two genes that causes prostate cancer in men and it is autosomal dominant. C. Studies have shown that the presence of the TP53 gene strongly influences the incidence of prostate cancer. D. Recent research has demonstrated that prostate cancer is the result of lifestyle factors and that genetics are unrelated.

Ans: A Rationale: A number of studies have identified an association of BRCA-2 mutation with an increased risk of prostate cancer.

A client's most recent blood work reveals low levels of albumin. This assessment finding should suggest the possibility of what nursing diagnosis? A. Risk for imbalanced fluid volume related to low albumin B. Risk for infection related to low albumin C. Ineffective tissue perfusion related to low albumin D. Impaired skin integrity related to low albumin

Ans: A Rationale: Albumin is particularly important for the maintenance of fluid balance within the vascular system. Deficiencies nearly always manifest as fluid imbalances. Tissue oxygenation and skin integrity are not normally affected. Low albumin does not constitute a risk for infection.

An older adult client has been diagnosed with aortic regurgitation. Which change in blood flow should the nurse expect to see on this client's echocardiogram? A. Blood to flow back from the aorta to the left ventricle B. Obstruction of blood flow from the left ventricle C. Blood to flow back from the left atrium to the left ventricle D. Obstruction of blood from the left atrium to left ventricle

Ans: A Rationale: Aortic regurgitation occurs when the aortic valve does not completely close, and blood flows back to the left ventricle from the aorta during diastole. Aortic regurgitation does not cause obstruction of blood flow from the left ventricle, blood to flow back from the left atrium to the left ventricle, or obstruction of blood from the left atrium to left ventricle.

A nurse is caring for a client who has a diagnosis of acute myelocytic leukemia (AML). Assessment of which factor most directly addresses the most common cause of death among clients with leukemia? A. Infection status B. Nutritional status C. Electrolyte levels D. Liver function

Ans: A Rationale: Because of the lack of mature and normal granulocytes that help fight infection, clients with leukemia are prone to infection. In clients with AML, death typically occurs from infection or bleeding. Symptoms of AML include weight loss, fever, night sweats, and fatigue, which would guide the nurse to monitor the client's nutrition and electrolytes. Gastrointestinal problems (nausea and vomiting) and electrolyte imbalances (hyperkalemia and hypocalcemia) may result from chemotherapy use. The liver is responsible for metabolism and metabolic detoxification, so monitoring liver function is important for the client who is receiving chemotherapy. These problems may contribute to and/or result in death but are not the most common cause.

A client is receiving treatment for a new diagnosis of chronic lymphocytic leukemia (CLL). Based on known risk factors, age, ethnicity, and accompanying clinical conditions, which client is most likely to have this disease? A. 82-year-old Vietnam War veteran with widely disseminated shingles B. 62-year-old client of Asian descent with a left fractured hip C. 69-year-old Gulf War veteran with deep vein thrombosis (DVT) D. 85-year-old client of Native American/First Nation descent with chest pain

Ans: A Rationale: CLL is a common malignancy of older adults with an average age of 71 at diagnosis and the most prevalent leukemia in the Western world. It is rarely seen in clients of Native American/First Nation descent and has an infrequent incidence in clients of Asian descent. Veterans of the Vietnam War who were exposed to the herbicide Agent Orange are at risk for CLL. The time period of exposure was from 1962 to 1975 so veterans from the Gulf War in 1991 were not exposed. Infections are common with advanced CLL. None of the other conditions are related to infection, so they are not the best choice. Viral infections such as herpes zoster (shingles) can be widely disseminated with CLL.

A postoperative cardiac client experiences signs and symptoms of cardiac tamponade. Which action by the nurse would be most appropriate? A. Prepare to assist with pericardiocentesis. B. Reposition the client into a prone position. C. Administer a dose of metoprolol as prescribed. D. Administer a bolus of normal saline as prescribed.

Ans: A Rationale: Cardiac tamponade requires immediate pericardiocentesis. Beta-blockers and fluid boluses will not relieve the pressure on the heart and prone positioning would likely exacerbate symptoms.

A 22-year-old male client is being discharged home after surgery for testicular cancer. The client is scheduled to begin chemotherapy in 2 weeks. The client tells the nurse that they do not think he can take weeks or months of chemotherapy, stating that they have researched the adverse effects online. What is the most appropriate nursing action for this client at this time? A. Provide empathy and encouragement in an effort to foster a positive outlook. B. Tell the client it is their decision whether to have chemotherapy. C. Report the client's statement to members of their support system. D. Refer the client to social work.

Ans: A Rationale: Clients may be required to endure a long course of therapy and will need encouragement to maintain a positive attitude. It is certainly the client's ultimate decision to accept or reject chemotherapy, but the nurse should focus on promoting a positive outlook. It would be a violation of confidentiality to report the client's statement to members of their support system and there is no obvious need for a social work referral.

A client who has undergone valve replacement surgery is being prepared for discharge home. Because the client will be discharged with a prescription for warfarin, the nurse would educate the client about the need to take which action? A. Undergo regular testing of the International Normalized Ratio (INR). B. Sleep in a semi-Fowler position for the first 6 to 8 weeks to prevent emboli. C. Avoid foods that contain vitamin K. D. Take enteric-coated acetylsalicylic acid (ASA) on a daily basis.

Ans: A Rationale: Clients who take warfarin after valve replacement have individualized target INRs; usually between 2 and 3.5 for mitral valve replacement and 1.8 and 2.2 for aortic valve replacement. Natural sources of vitamin K do not normally need to be avoided and ASA is not indicated. Sleeping upright is unnecessary.

A nurse is planning the care of client who has been diagnosed with essential thrombocythemia (ET). Which nursing diagnosis should the nurse prioritize when choosing interventions? A. Risk for ineffective tissue perfusion B. Risk for imbalanced fluid volume C. Risk for ineffective breathing pattern D. Risk for ineffective thermoregulation

Ans: A Rationale: Clients with ET are at risk for hypercoagulation and consequent ineffective tissue perfusion. Fluid volume, breathing, and thermoregulation are not normally affected.

A young student is brought to the school nurse after falling off a swing. The nurse is documenting that the child has bruising on the lateral aspect of the right arm. What term will the nurse use to describe bruising on the skin in documentation? A. Telangiectasias B. Ecchymoses C. Purpura D. Urticaria

Ans: B Rationale: Telangiectasias consist of red marks on the skin caused by stretching of superficial blood vessels. Ecchymoses are bruises, and purpura consists of pinpoint hemorrhages into the skin. Urticaria is wheals or hives.

A client with a history of cirrhosis is admitted to the ICU with a diagnosis of bleeding esophageal varices; an attempt to stop the bleeding has been only partially successful. What would the critical care nurse expect the care team to prescribe for this client? A. Packed red blood cells (PRBCs) B. Vitamin K C. Oral anticoagulants D. Heparin infusion

Ans: A Rationale: Clients with liver dysfunction may have life-threatening hemorrhage from peptic ulcers or esophageal varices. In these cases, replacement with fresh-frozen plasma, PRBCs, and platelets is usually required. Vitamin K may be prescribed once the bleeding is stopped, but that is not what is needed to stop the bleeding of the varices. Anticoagulants would exacerbate the client's bleeding.

A nurse is providing care for a client who has recently been admitted to the postsurgical unit from PACU following a transurethral resection of the prostate. The nurse is aware of the nursing diagnosis of Risk for Imbalanced Fluid Volume. In order to assess for this risk, the nurse should prioritize what action? A. Closely monitoring the input and output of the bladder irrigation system B. Administering parenteral nutrition and fluids as prescribed C. Monitoring the client's level of consciousness and skin turgor D. Scanning the client's bladder for retention every 2 hours

Ans: A Rationale: Continuous bladder irrigation effectively reduces the risk of clots in the GU tract but also creates a risk for fluid volume excess if it becomes occluded. The nurse must carefully compare input and output, and ensure that these are in balance. Parenteral nutrition is unnecessary after prostate surgery and skin turgor is not an accurate indicator of fluid status. Frequent bladder scanning is not required when a urinary catheter is in situ.

The cardiac nurse is caring for a client who has been diagnosed with dilated cardiomyopathy (DCM). Echocardiography is likely to reveal what pathophysiological finding? A. Decreased ejection fraction B. Decreased heart rate C. Ventricular hypertrophy D. Mitral valve regurgitation

Ans: A Rationale: DCM is distinguished by significant dilation of the ventricles without simultaneous hypertrophy. The ventricles have elevated systolic and diastolic volumes, but a decreased ejection fraction. Bradycardia and mitral valve regurgitation do not typically occur in clients with DCM.

A client with a recent diagnosis of ITP has asked the nurse why the care team has not chosen to administer platelets, stating, "I have low platelets, so why not give me a transfusion of exactly what I'm missing?" How should the nurse best respond? A. "Transfused platelets usually aren't beneficial because they're rapidly destroyed in the body." B. "A platelet transfusion often further blunts your body's own production of platelets." C. "Finding a matching donor for a platelet transfusion is exceedingly difficult." D. "A very small percentage of the platelets in a transfusion are actually functional."

Ans: A Rationale: Despite extremely low platelet counts, platelet transfusions are usually avoided. Transfusions tend to be ineffective not because the platelets are nonfunctional but because the client's antiplatelet antibodies bind with the transfused platelets, causing them to be destroyed. Matching the client's blood type is not usually necessary for a platelet transfusion. Platelet transfusions do not exacerbate low platelet production.

When assessing venous disease in a client's lower extremities, the nurse knows that what test will most likely be prescribed? A. Duplex ultrasonography B. Echocardiography C. Positron emission tomography (PET) D. Radiography

Ans: A Rationale: Duplex ultrasound may be used to determine the level and extent of venous disease as well as its chronicity. Radiographs (x-rays), PET scanning, and echocardiography are never used for this purpose as they do not allow visualization of blood flow.

The nurse is caring for a client with a history of endocarditis. Which topic would the nurse prioritize during health promotion education? A. Oral hygiene B. Physical activity C. Dietary guidelines D. Fluid intake

Ans: A Rationale: For clients with endocarditis, regular professional oral care combined with personal oral care may reduce the risk of bacteremia. In most cases, diet and fluid intake do not need to be altered. Physical activity has broad benefits, but it does not directly prevent complications of endocarditis.

A client has just returned to the floor following a transurethral resection of the prostate. A triple-lumen indwelling urinary catheter has been inserted for continuous bladder irrigation. What, in addition to balloon inflation, are the functions of the three lumens? A. Continuous inflow and outflow of irrigation solution B. Intermittent inflow and continuous outflow of irrigation solution C. Continuous inflow and intermittent outflow of irrigation solution D. Intermittent flow of irrigation solution and prevention of hemorrhage

Ans: A Rationale: For continuous bladder irrigation, a triple-lumen indwelling urinary catheter is inserted. The three lumens provide for balloon inflation and continuous inflow and outflow of irrigation solution.

Which intervention is the most effective way to prevent rheumatic heart disease? A. Recognizing and promptly treating streptococcal infections B. Prophylactic use of calcium channel blockers in high-risk populations C. Adhering closely to the recommended child immunization schedule D. Promoting smoking cessation in all clients who smoke

Ans: A Rationale: Group A streptococcus can cause rheumatic heart fever, resulting in rheumatic endocarditis. Being aware of signs and symptoms of streptococcal infections, identifying them quickly, and treating them promptly are the best preventative techniques for rheumatic endocarditis. Smoking cessation, immunizations, and calcium channel blockers will not prevent rheumatic heart disease.

Which classification of clients would be at greatest risk for hospital-acquired endocarditis? A. Hemodialysis clients B. Clients on immunoglobulins C. Clients who undergo intermittent urinary catheterization D. Children under the age of 12

Ans: A Rationale: Hospital-acquired infective endocarditis occurs most often in clients with debilitating disease or indwelling catheters and in clients who are receiving hemodialysis or prolonged IV fluid or antibiotic therapy. Clients taking immunosuppressive medications or corticosteroids are more susceptible to fungal endocarditis. Clients on immunoglobulins, those who need in and out catheterization, and children are not at increased risk for nosocomial infective endocarditis.

The nurse is caring for a client who is scheduled to undergo mechanical valve replacement. Client education would include coverage of which intervention? A. Use of client-controlled analgesia B. Long-term anticoagulant therapy C. Steroid therapy D. Use of IV diuretics

Ans: B Rationale: Mechanical valves necessitate long-term use of required anticoagulants. Diuretics and steroids are not indicated and client-controlled analgesia may or may be not be used in the immediate postoperative period.

An oncology nurse is caring for a client with multiple myeloma who is experiencing bone destruction. When reviewing the client's most recent blood tests, the nurse should anticipate which imbalance? A. Hypercalcemia B. Hyperproteinemia C. Elevated serum viscosity D. Elevated red blood count (RBC)

Ans: A Rationale: Hypercalcemia may result when bone destruction occurs due to the disease process. Elevated serum viscosity occurs because plasma cells excrete excess immunoglobulin but would not result from bone destruction. The RBC count will decrease, not increase, resulting in anemia due to the abnormal protein produced from the malignant cells. Hyperproteinemia is defined as high protein in the blood and is commonly seen in clients with dehydration but would not result from bone destruction.

A client with leukemia has developed stomatitis and is experiencing a nutritional deficit. An oral anesthetic has consequently been prescribed. What health education should the nurse provide to the client? A. Chew with care to avoid inadvertently biting the tongue. B. Use the oral anesthetic 1 hour prior to mealtime. C. Brush teeth before and after eating. D. Swallow slowly and deliberately.

Ans: A Rationale: If oral anesthetics are used, the client must be warned to chew with extreme care to avoid inadvertently biting the tongue or buccal mucosa. An oral anesthetic would be metabolized by the time the client eats if it is used 1 hour prior to meals. There is no specific need to warn the client about brushing teeth or swallowing slowly because an oral anesthetic has been used.

A client with advanced venous insufficiency is confined to bed rest following orthopedic surgery. How can the nurse best prevent skin breakdown in the client's lower extremities? A. Ensure that the client's heels are protected and supported. B. Closely monitor the client's serum albumin and prealbumin levels. C. Perform gentle massage of the client's lower legs, as tolerated. D. Perform passive range-of-motion exercises once per shift.

Ans: A Rationale: If the client is on bed rest, it is important to relieve pressure on the heels to prevent pressure ulcerations, since the heels are among the most vulnerable body regions. Monitoring blood work does not directly prevent skin breakdown, even though albumin is related to wound healing. Massage is not normally indicated and may exacerbate skin breakdown. Passive range-of-motion exercises do not directly reduce the risk of skin breakdown.

A client with a diagnosis of acute myeloid leukemia (AML) is being treated with induction therapy on the oncology unit. What nursing action should be prioritized in the client's care plan? A. Protective isolation and vigilant use of standard precautions B. Provision of a high-calorie, low-texture diet and appropriate oral hygiene C. Including the family in planning the client's activities of daily living D. Monitoring and treating the client's pain

Ans: A Rationale: Induction therapy causes neutropenia and a severe risk of infection. This risk must be addressed directly in order to ensure the client's survival. For this reason, infection control would be prioritized over nutritional interventions, family care, and pain, even though each of these are important aspects of nursing care.

An older adult client is exhibiting many of the characteristic signs and symptoms of iron deficiency. In addition to a complete blood count, what diagnostic assessment should the nurse anticipate? A. Stool for occult blood B. Bone marrow biopsy C. Lumbar puncture D. Urinalysis

Ans: A Rationale: Iron deficiency in the adult generally indicates blood loss (e.g., from bleeding in the GI tract or heavy menstrual flow). Bleeding in the GI tract can be preliminarily identified by testing stool for the presence of blood. A bone marrow biopsy would not be undertaken for the sole purpose of investigating an iron deficiency. Lumbar puncture and urinalysis would not be clinically relevant.

A client who is postoperative day 12 and recovering at home following a laparoscopic prostatectomy has reported that the client is experiencing occasional "dribbling" of urine. How should the nurse best respond to this client's concern? A. Inform the client that urinary control is likely to return gradually. B. Arrange for the client to be assessed by the urologist. C. Facilitate the insertion of an indwelling urinary catheter by the home care nurse. D. Teach the client to perform intermittent self-catheterization.

Ans: A Rationale: It is important that the client know that regaining urinary control is a gradual process; the client may continue to "dribble" after being discharged from the hospital, but this should gradually diminish (usually within 1 year). At this point, medical follow-up is likely not necessary. There is no need to perform urinary catheterization.

An oncology nurse is providing health education for a client who has recently been diagnosed with leukemia. What should the nurse explain about commonalities between all of the different subtypes of leukemia? A. The different leukemias all involve unregulated proliferation of white blood cells. B. The different leukemias all have unregulated proliferation of red blood cells and decreased bone marrow function. C. The different leukemias all result in a decrease in the production of white blood cells. D. The different leukemias all involve the development of cancer in the lymphatic system.

Ans: A Rationale: Leukemia commonly involves unregulated proliferation of white blood cells. Decreased production of red blood cells is associated with anemias. Decreased production of white blood cells is associated with leukopenia. The leukemias are not characterized by their involvement with the lymphatic system.

The nurse is caring for a client who is seeking care for signs and symptoms of lymphedema. The nurse's plan of care should prioritize which nursing diagnosis? A. Risk for infection related to lower extremity swelling secondary to lymphedema B. Disturbed body image related to lower extremity swelling secondary to lymphedema C. Ineffective health maintenance related to lower extremity swelling secondary to lymphedema D. Risk for deficient fluid volume related to lower extremity swelling secondary to lymphedema

Ans: A Rationale: Lymphed ema, which is caused by accumulation of lymph in the tissues, constitutes a significant risk for infection. The client's body image is likely to be disturbed, and the nurse should address this, but infection is a more significant threat to the client's physiologic well-being. Lymphedema is unrelated to ineffective health maintenance and deficient fluid volume is not a significant risk.

A 75-year-old male client is being treated for phimosis. When planning this client's care, what health promotion activity is most directly related to the etiology of the client's health problem? A. Teaching the client about safer sexual practices B. Teaching the client about the importance of hygiene C. Teaching the client about the safe use of PDE-5 inhibitors D. Teaching the client to perform testicular self-examination

Ans: B Rationale: Poor hygiene often contributes to cases of phimosis. This health problem is unrelated to sexual practices, the use of PDE-5 inhibitors, or testicular self-examination.

Following an extensive diagnostic workup, a client has been diagnosed with myelodysplastic syndrome (MDS). Which assessment question most directly addresses the potential etiology of this client's health problem? A. "Were you ever exposed to toxic chemicals in any of the jobs that you held?" B. "When you were younger, did you tend to have recurrent infections of any kind?" C. "Have you ever smoked cigarettes or used other tobacco products?" D. "Would you say that you've had a lot of sun exposure in your lifetime?"

Ans: A Rationale: MDS is idiopathic in nature due to HSC damage, although 10% to 15% of clients will develop MDS following exposure to alkylating agents, radiotherapy, or chemicals (e.g., benzene), and/or have an inherited genetic disorder, such as Fanconi anemia or trisomy 21. Genetic syndromes account for about 50% of cases (e.g., Down syndrome, trisomy 8 syndrome, neurofibromatosis type 1). MDS is not known to be caused by an infection, tobacco use, or sun exposure.

A nurse is creating an education plan for a client with venous insufficiency. Which measure should the nurse include in the plan? A. Avoid normal stockings that are tight. B. Limit activities, including walking. C. Sleep with legs below heart level. D. Refrain from using graduated compression stockings.

Ans: A Rationale: Measures taken to prevent complications include avoiding tight-fitting socks and panty girdles; maintaining activities, such as walking; sleeping with legs elevated; and using pressure stockings. Not included in the teaching plan for venous insufficiency would be reducing activity, sleeping with legs dependent, and avoiding pressure stockings. Each of these actions exacerbates venous insufficiency.

A 60-year-old client with chronic myeloid leukemia (CML) will be treated in the home setting, and the nurse is preparing appropriate health education. Which topic should the nurse emphasize? A. The importance of adhering to the prescribed drug regimen B. The need to ensure that vaccinations are up to date C. The importance of daily physical activity D. The need to avoid shellfish and raw foods

Ans: A Rationale: Nurses need to understand that the effectiveness of the drugs used to treat CML is based on the ability of the client to adhere to the medication regimen as prescribed. Adherence is often incomplete, thus this must be a focus of health education. Vaccinations normally would not be given during treatment, and daily physical activity may be impossible for the client. Dietary restrictions are not normally necessary.

The nurse is admitting a 32-year-old woman to the presurgical unit. The nurse learns during the admission assessment that the client takes oral contraceptives. The nurse's postoperative plan of care should include what intervention? A. Early ambulation and leg exercises B. Cessation of the oral contraceptives until 3 weeks' postoperative C. Doppler ultrasound of peripheral circulation twice daily D. Dependent positioning of the client's extremities when at rest

Ans: A Rationale: Oral contraceptive use increases blood coagulability; with bed rest, the client may be at increased risk of developing deep vein thrombosis. Leg exercises and early ambulation are among the interventions that address this risk. Assessment of peripheral circulation is important, but Doppler ultrasound may not be necessary to obtain these data. Dependent positioning increases the risk of venous thromboembolism (VTE). Contraceptives are not normally discontinued to address the risk of VTE in the short term.

The nurse is preparing a client for cardiac surgery. During the procedure, the client's heart will be removed and a donor heart implanted at the vena cava and pulmonary veins. What procedure will this client undergo? A. Orthotopic transplant B. Xenograft C. Heterotopic transplant D. Homograft

Ans: A Rationale: Orthotopic transplantation is the most common surgical procedure for cardiac transplantation. The recipient's heart is removed, and the donor heart is implanted at the vena cava and pulmonary veins. Some surgeons still prefer to remove the recipient's heart, leaving a portion of the recipient's atria (with the vena cava and pulmonary veins) in place. Homografts, or allografts (i.e., human valves), are obtained from cadaver tissue donations and are used for aortic and pulmonic valve replacement. Xenografts and heterotopic transplantation are not terms used to describe heart transplantation.

A client has experienced occasional urinary incontinence in the weeks since their prostatectomy. In order to promote continence, the nurse should encourage which of the following? A. Pelvic floor exercises B. Intermittent urinary catheterization C. Reduced physical activity D. Active range of motion exercises

Ans: A Rationale: Pelvic floor muscles can promote the resumption of normal urinary function following prostate surgery. Catheterization is normally unnecessary, and it carries numerous risks of adverse effects. Increasing or decreasing physical activity is unlikely to influence urinary function.

The nurse's brief review of a client's electronic health record indicates that the client regularly undergoes therapeutic phlebotomy. Which of the following rationales for this procedure is most plausible? A. The client may chronically produce excess red blood cells. B. The client may frequently experience a low relative plasma volume. C. The client may have impaired stem cell function. D. The client may previously have undergone bone marrow biopsy.

Ans: A Rationale: Persistently elevated hematocrit is an indication for therapeutic phlebotomy. It is not used to address excess or deficient plasma volume and is not related to stem cell function. Bone marrow biopsy is not an indication for therapeutic phlebotomy.

A client suffers a leg wound which causes minor blood loss. As a result of bleeding, the process of primary hemostasis is activated. What will occur during this process? A. Severed blood vessels constrict. B. Thromboplastin is released. C. Prothrombin is converted to thrombin. D. Fibrin is lysed.

Ans: A Rationale: Primary hemostasis involves the severed vessel constricting and platelets collecting at the injury site. Secondary hemostasis occurs when thromboplastin is released, prothrombin converts to thrombin, and fibrin is lysed.

A 35-year-old man is seen in the clinic because the client is experiencing recurring episodes of urinary frequency, dysuria, and fever. The nurse should recognize the possibility of what health problem? A. Chronic bacterial prostatitis B. Orchitis C. Benign prostatic hyperplasia D. Urolithiasis

Ans: A Rationale: Prostatitis is an inflammation of the prostate gland that is often associated with lower urinary tract symptoms and symptoms of sexual discomfort and dysfunction. Symptoms are usually mild, consisting of frequency, dysuria, and occasionally urethral discharge. Urinary incontinence and retention occur with benign prostatic hyperplasia or hypertrophy. The client may experience nocturia, urgency, and decrease in volume and force of urinary stream. Urolithiasis is characterized by excruciating pain. Orchitis does not cause urinary symptoms.

A client's absolute neutrophil count (ANC) is 440/mm3 but the nurse's assessment reveals no apparent signs or symptoms of infection. What action should the nurse prioritize when providing care for this client? A. Meticulous hand hygiene B. Timely administration of antibiotics C. Provision of a nutrient-dense diet D. Maintaining a sterile care environment

Ans: A Rationale: Providing care for a client with neutropenia requires that the nurse adhere closely to standard precautions and infection control procedures. Hand hygiene is central to such efforts. Prophylactic antibiotics are rarely used and it is not possible to provide a sterile environment for care. Nutrition is highly beneficial, but hand hygiene is the central aspect of care.

A client is admitted to the critical care unit with a diagnosis of cardiomyopathy. When reviewing the client's most recent laboratory results, the nurse would prioritize assessment of which value? A. Sodium B. Aspartate aminotransferase, alanine aminotransferase, and bilirubin C. White blood cell differential D. Blood urea nitrogen (BUN)

Ans: A Rationale: Sodium is the major electrolyte involved with cardiomyopathy. Cardiomyopathy often leads to heart failure which develops, in part, from fluid overload. Fluid overload is often associated with elevated sodium levels. Consequently, sodium levels are followed more closely than other important laboratory values, including BUN, leukocytes, and liver function tests.

An 83-year-old client has been prescribed finasteride. When performing client education with this client, the nurse should be sure to tell the client to take what action? A. Report the planned use of dietary supplements to the healthcare provider. B. Decrease the intake of fluids to prevent urinary retention. C. Abstain from sexual activity for 2 weeks following the initiation of treatment. D. Anticipate a temporary worsening of urinary retention before symptoms subside.

Ans: A Rationale: Some herbal supplements are contraindicated with finasteride, thus their planned use should be discussed with the health care provider or pharmacist. The client should maintain normal fluid intake. There is no need to abstain from sexual activity and a worsening of urinary retention is not anticipated.

A client has been diagnosed with a valvular disorder. The client tells the nurse that the client has read about numerous treatment options, including valvuloplasty. Which statement would be most appropriate for the nurse to make regarding valvuloplasty? A. "For some clients, valvuloplasty can be done in a cardiac catheterization laboratory." B. "Valvuloplasty is a dangerous procedure, but it has excellent potential if it goes well." C. "Valvuloplasty is open heart surgery, but this is very safe these days and normally requires only an overnight hospital stay." D. "It's prudent to get a second opinion before deciding to have valvuloplasty."

Ans: A Rationale: Some valvuloplasty procedures do not require general anesthesia or cardiopulmonary bypass and can be performed in a cardiac catheterization laboratory or hybrid room. Open heart surgery is not required and the procedure does not carry exceptional risks that would designate it as being dangerous. Normally there is no need for the nurse to advocate for a second opinion.

A client is receiving a blood transfusion and reports a new onset of slight dyspnea. The nurse's rapid assessment reveals bilateral lung crackles and elevated BP. What is the nurse's most appropriate action? A. Slow the infusion rate and monitor the client closely. B. Discontinue the transfusion and begin resuscitation. C. Pause the transfusion and administer a 250 mL bolus of normal saline. D. Discontinue the transfusion and administer a beta-blocker, as prescribed.

Ans: A Rationale: The client is showing early signs of hypervolemia; the nurse should slow the infusion rate and assess the client closely for any signs of exacerbation. At this stage, discontinuing the transfusion is not necessary. A bolus would worsen the client's fluid overload.

The clinic nurse is caring for a 57-year-old client who reports experiencing leg pain whenever walking several blocks. The client has type 1 diabetes and has smoked a pack of cigarettes every day for the past 40 years. The health care provider diagnoses intermittent claudication. The nurse should provide which instruction about long-term care to the client? A. "Be sure to practice meticulous foot care." B. "Consider cutting down on your smoking." C. "Reduce your activity level to accommodate your limitations." D . "Try to make sure you eat enough protein."

Ans: A Rationale: The client with peripheral vascular disease or diabetes should receive education or reinforcement about skin and foot care. Intermittent claudication and other chronic peripheral vascular diseases reduce oxygenation to the feet, making them susceptible to injury and poor healing; therefore, meticulous foot care is essential. The client should stop smoking—not just cut down—because nicotine is a vasoconstrictor. Daily walking benefits the client with intermittent claudication. Increased protein intake will not alleviate the client's symptoms.

A 29-year-old client has just been told that they have testicular cancer and needs to have surgery. During a presurgical appointment, the client admits to feeling devastated that they require surgery, stating that it will leave them "emasculated" and "a shell of a man." The nurse should identify what nursing diagnosis when planning the client's subsequent care? A. Disturbed body image related to effects of surgery B. Spiritual distress related to effects of cancer surgery C. Social isolation related to effects of surgery D. Risk for loneliness related to change in self-concept

Ans: A Rationale: The client's statements specifically address the perception of the body as it relates to the client's identity. Consequently, a nursing diagnosis of Disturbed Body Image is likely appropriate. This client is at risk for social isolation and loneliness, but there's no indication in this scenario that these diagnoses are present. There is no indication of a spiritual element to the client's concerns.

A client is scheduled to undergo a bone marrow aspiration. When preparing the client for the procedure, which action would the nurse do first? A. Ensure informed consent has been obtained. B. Cleanse the skin with an antiseptic. C. Administer a local anesthetic. D. Cover the area with a sterile drape.

Ans: A Rationale: The first step in the procedure is ensuring that informed consent has been obtained by the health care provider, nurse practitioner, or health care provider assistant performing the procedure and includes the reason the procedure is being performed, alternatives, and risks of the procedure. Risks include infection, bleeding, and pain. After informed consent is obtained, the client is assisted to either a prone or lateral decubitus position. The skin is cleansed using aseptic technique and either a chlorhexidine-based solution or povidone-iodine. A sterile drape is applied, and the skin is numbed using local anesthesia.

A nurse is teaching a 53-year-old man about prostate cancer, given the fact that the client has a family history of the disease. What information should the nurse provide to best facilitate the early identification of prostate cancer? A. Have a digital rectal examination and prostate-specific antigen (PSA) test done as recommended. B. Have a transrectal ultrasound every 5 years. C. Perform monthly testicular self-examinations, especially after age 60. D. Have a complete blood count (CBC), blood urea nitrogen (BUN), and creatinine assessment performed annually.

Ans: A Rationale: The incidence of prostate cancer increases after age 50. The digital rectal examination, which identifies enlargement or irregularity of the prostate, and the PSA test, a tumor marker for prostate cancer, are effective diagnostic measures that are especially relevant when a client has a family history.

A nurse is assessing a client who presented to the ED with priapism. The student nurse is aware that this condition is classified as a urologic emergency because of the potential for what issue? A. Permanent vascular damage B. Chronic pain C. Urinary tract infection D. Future erectile dysfunction

Ans: A Rationale: The ischemic form of priapism, which is described as a nonsexual, persistent erection with little or no cavernous blood flow, must be treated promptly to prevent permanent damage to the penis.

The nurse is preparing to administer a unit of platelets to an adult client. When administering this blood product, which of the following actions should the nurse perform? A. Administer the platelets as rapidly as the client can tolerate. B. Establish IV access as soon as the platelets arrive from the blood bank. C. Ensure that the client has a patent central venous catheter. D. Aspirate 10 to 15 mL of blood from the client's IV immediately following the transfusion.

Ans: A Rationale: The nurse should infuse each unit of platelets as fast as client can tolerate to diminish platelet clumping during administration. IV access should be established prior to obtaining the platelets from the blood bank. A central line is appropriate for administration, but peripheral IV access (22-gauge or larger) is sufficient. There is no need to aspirate after the transfusion.

A client who has undergone a femoral to popliteal bypass graft surgery returns to the surgical unit. Which assessments should the nurse perform during the first postoperative day? A. Assess pulse of affected extremity every 15 minutes at first. B. Palpate the affected leg for pain during every assessment. C. Assess the client for signs and symptoms of compartment syndrome every 2 hours. D. Perform Doppler evaluation once daily.

Ans: A Rationale: The primary objective in the postoperative period is to maintain adequate circulation through the arterial repair. Pulses, Doppler assessment, color and temperature, capillary refill, and sensory and motor function of the affected extremity are checked and compared with those of the other extremity; these values are recorded initially every 15 minutes and then at progressively longer intervals if the client's status remains stable. Doppler evaluations should be performed every 2 hours. Pain is regularly assessed, but palpation is not the preferred method of performing this assessment. Compartment syndrome results from the placement of a cast, not from vascular surgery.

The nurse is planning care for a client with venous insufficiency. Which nursing intervention would be appropriate for this client's plan of care? A. Elevate lower extremities. B. Educate on decreased protein. C. Apply compression only at night. D. Teach frequent rest periods due to pain.

Ans: A Rationale: Venous insufficiency is lack of blood flow back to the heart. Elevation of lower extremities will assist the peripheral blood vessels in returning stasis of blood. Increased protein should be taught. Compression therapy should be used but not only at night. Pain is not usually assessed in clients with venous insufficiency but with arterial insufficiency.

A client has been living with dilated cardiomyopathy for several years but has experienced worsening symptoms despite aggressive medical management. The nurse would anticipate which potential treatment? A. Heart transplantation B. Balloon valvuloplasty C. Cardiac catheterization D. Stent placement

Ans: A Rationale: When heart failure progresses and medical treatment is no longer effective, surgical intervention, including heart transplantation, is considered. Valvuloplasty, stent placement, and cardiac catheterization will not address the pathophysiology of cardiomyopathy.

The nurse is caring for a client with a large venous leg ulcer. What intervention should the nurse implement to promote healing and prevent infection? A. Provide a high-calorie, high-protein diet. B. Apply a clean occlusive dressing once daily and whenever soiled. C. Abstain from wearing graduated compression stockings. D. Apply an antibiotic ointment on the surrounding skin with each dressing change.

Ans: A Rationale: Wound healing is highly dependent on adequate nutrition. The diet should be sufficiently high in calories and protein. Antibiotic ointments are not normally used on the skin surrounding a leg ulcer and occlusive dressings can exacerbate impaired blood flow. Compression therapy should be implemented with venous ulcers but not arterial ulcers.

The nurse is leading a workshop on sexual health for men. The nurse should describe what organic causes of erectile dysfunction? Select all that apply. A. Diabetes B. Testosterone deficiency C. Anxiety D. Depression E. Parkinsonism

Ans: A, B, E Rationale: Organic causes of ED include cardiovascular disease, endocrine disease (diabetes, pituitary tumors, testosterone deficiency, hyperthyroidism, and hypothyroidism), cirrhosis, chronic kidney disease, genitourinary conditions (radical pelvic surgery), hematologic conditions (Hodgkin disease, leukemia), neurologic disorders (neuropathies, parkinsonism, spinal cord injury, multiple sclerosis), trauma to the pelvic or genital area, alcohol, smoking, medications, and drug abuse. Anxiety and depression are considered to be psychogenic causes.

The results of a client's most recent blood work and physical assessment are suggestive of immune thrombocytopenic purpura (ITP). This client should undergo testing for which of the following potential causes? Select all that apply. A. Hepatitis B. Acute kidney injury C. HIV D. Malignant melanoma E. Cholecystitis

Ans: A, C Rationale: Viral illnesses have the potential to cause ITP. Acute kidney injury, malignancies, and gallbladder inflammation are not typical causes of ITP.

A client has come to the OB/GYN clinic due to recent heavy menstrual flow. Because of the client's consequent increase in red cell production, the nurse should recommend the client increase daily intake of what substance? A. Vitamin E B. Vitamin D C. Iron D. Magnesium

Ans: C Rationale: To replace blood loss, the rate of red cell production increases. Iron is incorporated into hemoglobin. Vitamins E and D and magnesium do not need to be increased when RBC production is increased.

A 76-year-old with a diagnosis of penile cancer has been admitted to the medical floor. Because the incidence of penile cancer is so low, the staff educator has been asked to teach about penile cancer. What risk factors should the educator cite in this presentation? Select all that apply. A. Phimosis B. Priapism C. Herpes simplex infection D. Increasing age E. Lack of circumcision

Ans: A, D, E Rationale: Several risk factors for penile cancer have been identified, including lack of circumcision, poor genital hygiene, phimosis, HPV, smoking, ultraviolet light treatment of psoriasis on the penis, increasing age (two thirds of cases occur in men older than 65 years of age), lichen sclerosus, and balanitis xerotica obliterans. Priapism and HSV are not known risk factors.

The client has just returned to the floor after balloon valvuloplasty of the aortic valve and the nurse is planning appropriate assessments. The nurse would assess for indications of which potential complications? Select all that apply. A. Emboli B. Mitral valve damage C. Ventricular dysrhythmia D. Atrial-septal defect E. Plaque formation

Ans: A,B,C Rationale: Possible complications include aortic regurgitation, emboli, ventricular perforation, rupture of the aortic valve annulus, ventricular dysrhythmia, mit ral valve damage, and bleeding from the catheter insertion sites. Atrial-septal defect and plaque formation are not complications of a balloon valvuloplasty.

A nurse is writing the care plan of a client who has been diagnosed with myelofibrosis. What nursing diagnoses should the nurse address? Select all that apply. A. Disturbed body image B. Impaired mobility C. Imbalanced nutrition: Less than body requirements D. Acute confusion E. Risk for infection

Ans: A,B,C,E Rationale: The profound splenomegaly that accompanies myelofibrosis can impact the client's body image and mobility. As well, nutritional deficits are common and the client is at risk for infection. Cognitive effects are less common.

The nurse is evaluating a client's diagnosis of arterial insufficiency with reference to the adequacy of the client's blood flow. On what physiologic variables does adequate blood flow depend? Select all that apply. A. Efficiency of heart as a pump B. Adequacy of circulating blood volume C. Ratio of platelets to red blood cells D. Size of red blood cells E. Patency and responsiveness of the blood vessels

Ans: A,B,E Rationale: Adequate blood flow depends on the efficiency of the heart as a pump, the patency and responsiveness of the blood vessels, and the adequacy of circulating blood volume. Adequacy of blood flow does not primarily depend on the size of red cells or their ratio to the number of platelets.

The nurse is caring for a client with mitral valve prolapse. Which symptoms would be consistent with this diagnosis? Select all that apply. A. Anxiety B. Fatigue C. Shoulder pain D. Tachypnea E. Palpitations

Ans: A,B,E Rationale: Most people who have mitral valve prolapse never have symptoms. A few have symptoms of fatigue, shortness of breath, lightheadedness, dizziness, syncope, palpitations, chest pain, and anxiety. Hyperpnea and shoulder pain are not characteristic symptoms of mitral valve prolapse.

The nurse has performed a thorough nursing assessment of the care of a client with chronic leg ulcers. The nurse's assessment should include which of the following components? Select all that apply. A. Location and type of pain B. Apical heart rate C. Bilateral comparison of peripheral pulses D. Comparison of temperature in the client's legs E. Identification of mobility limitations

Ans: A,C,D,E Rationale: A careful nursing history and assessment are important. The extent and type of pain are carefully assessed, as are the appearance and temperature of the skin of both legs. The quality of all peripheral pulses is assessed, and the pulses in both legs are compared. Any limitation of mobility and activity that results from vascular insufficiency is identified. It is not likely that there is any direct indication for assessment of apical heart rate, although peripheral pulses must be assessed.

An oncology nurse recognizes a client's risk for fluid imbalance while the client is undergoing treatment for leukemia. What related assessments should the nurse include in the client's plan of care? Select all that apply. A. Monitoring the client's electrolyte levels B. Monitoring the client's hepatic function C. Measuring the client's weight on a daily basis D. Measuring and recording the client's intake and output E. Auscultating the client's lungs frequently

Ans: A,C,D,E Rationale: Assessments that relate to fluid balance include monitoring the client's electrolytes, auscultating the client's chest for adventitious sounds, weighing the client daily, and closely monitoring intake and output. Liver function is not directly relevant to the client's fluid status in most cases.

Through the process of hematopoiesis, stem cells differentiate into either myeloid or lymphoid stem cells. Into what do myeloid stem cells further differentiate? Select all that apply. A. Leukocytes B. Natural killer cells C. Cytokines D. Platelets E. Erythrocytes

Ans: A,D,E Rationale: Myeloid stem cells differentiate into three broad cell types: erythrocytes, leukocytes, and platelets. Natural killer cells and cytokines do not originate as myeloid stem cells.

A client with a suspected malignant melanoma is referred to the dermatology clinic. The nurse knows to facilitate what diagnostic test to rule out a skin malignancy? A. Tzancksmear B. Skin biopsy C. Patch testing D. Skinscrapings

Ans: B Rationale: A skin biopsy is done to rule out malignancies of skin lesions. A Tzanck smear is used to examine cells from blistering skin conditions, such as herpes zoster. Patch testing is performed to identify substances to which the client has developed an allergy. Skin scrapings are done for suspected fungal infections.

A gerontologic nurse is teaching a group of nursing students about integumentary changes that occur in older adults. How should these students best integrate these changes into care planning? A. Byavoidingtheuseofmoisturizinglotionsonolderadults'skin B. By protecting older adults against shearing injuries C. By avoiding the use of ice packs to treat muscle pain D. Byprotectingolderadultsagainstexcessivesweataccumulation

Ans: B Rationale: Cellular changes associated with aging include thinning at the junction of the dermis and epidermis, which creates a risk for shearing injuries. Moisturizing lotions can be safely used to address the increased dryness of older adults' skin. Ice packs can be used, provided skin is assessed regularly and the client possesses normal sensation. Older adults perspire much less than younger adults, thus sweat accumulation is rarely an issue.

A nurse is doing a shift assessment on a group of clients after first taking report. An older adult client is having the second dose of IV antibiotics for a diagnosis of pneumonia. The nurse notices a new rash on the client's chest. The nurse should ask what priority question regarding the presence of a reddened rash? A. "Istherashworseataparticulartimeorseason?" B. "Are you allergic to any foods or medication?" C. "Are you having any loss of sensation in that area?" D. "Isyourrashpainful?"

Ans: B Rationale: The nurse should suspect an allergic reaction to the antibiotic therapy. Allergies can be a significant threat to the client's immediate health, thus questions addressing this possibility would be prioritized over those addressing sensation. Asking about previous rashes is important, but this should likely be framed in the context of an allergy assessment.

A wound care nurse is reviewing skin anatomy with a group of medical nurses. Which area of the skin would the nurse identify as providing a cushion between the skin layers, muscles, and bones? A. Dermis B. Subcutaneous tissue C. Epidermis D. Stratum corneum

Ans: B Rationale: The subcutaneous tissue, or hypodermis, is the innermost layer of the skin that is responsible for providing a cushion between the skin layers, muscles, and bones. The dermis is the largest portion of the skin, providing strength and structure. The epidermis is the outermost layer of stratified epithelial cells and composed of keratinocytes. The stratum corneum is the outermost layer of the epidermis, which provides a barrier to prevent epidermal water loss.

The nurse is taking a health history of a new client who reports pain in the left lower leg and foot when walking. This pain is relieved with rest, and the nurse observes that the left lower leg is slightly edematous and is hairless. When planning this client's care, the nurse should most likely address which health problem? A. Coronary artery disease (CAD) B. Intermittent claudication C. Arterial embolus D. Raynaud disease

Ans: B Rationale: A muscular, cramp-type pain in the extremities consistently reproduced with the same degree of exercise or activity and relieved by rest is experienced by clients with peripheral arterial insufficiency. Referred to as intermittent claudication, this pain is caused by the inability of the arterial system to provide adequate blood flow to the tissues in the face of increased demands for nutrients and oxygen during exercise. The nurse would not suspect the client has CAD, arterial embolus, or Raynaud disease; none of these health problems produce this cluster of signs and symptoms.

The nurse is auscultating the breath sounds of a client with pericarditis. Which finding is most consistent with this diagnosis? A. Wheezes B. Friction rub C. Fine crackles D. Coarse crackles

Ans: B Rationale: A pericardial friction rub is diagnostic of pericarditis. Crackles are associated with pulmonary edema and fluid accumulation, whereas wheezes signal airway constriction; neither of these occurs with pericarditis.

An occupational health nurse is providing an educational event and has been asked by an administrative worker about the risk of varicose veins. Which action should the nurse suggest as a preventive measure for varicose veins? A. Sit with crossed legs for a few minutes each hour to promote relaxation. B. Walk for several minutes every hour to promote circulation. C. Elevate the legs when tired. D. Wear snug-fitting ankle socks to decrease edema.

Ans: B Rationale: A proactive approach to preventing varicose veins would be to walk for several minutes every hour to promote circulation. Sitting with crossed legs may promote relaxation, but it is contraindicated for clients with, or at risk for, varicose veins. Elevating the legs only helps blood passively return to the heart and does not help maintain the competency of the valves in the veins. Wearing tight ankle socks is contraindicated for clients with, or at risk for, varicose veins; socks that are below the muscles of the calf do not promote venous return the socks simply capture the blood and promote venous stasis.

While assessing a client, the nurse notes that the client's ankle-brachial index (ABI) of the right leg is 0.40. How should the nurse best follow up this assessment finding? A. Assess the client's use of over-the-counter dietary supplements. B. Implement interventions relevant to arterial narrowing. C. Encourage the client to increase intake of foods high in vitamin K. D. Adjust the client's activity level to accommodate decreased coronary output.

Ans: B Rationale: ABI is used to assess the degree of stenosis of peripheral arteries. An ABI of less than 1.0 indicates possible claudication of the peripheral arteries. It does not indicate inadequate coronary output. There is no direct indication for changes in vitamin K intake and over-the-counter (OTC) medications are not likely causative.

A client has returned to the medical-surgical floor from the post-anesthesia care unit (PACU). Upon inspection, the nurse notes leakage around the suprapubic tube. Which action by the nurse is most appropriate? A. Cleanse the skin surrounding the suprapubic tube. B. Inform the urologist of this finding as it is abnormal. C. Remove the suprapubic tube and apply a wet to dry dressing. D. Administer antispasmodic drugs as prescribed.

Ans: B Rationale: After removal of the suprapubic catheter there may be urinary drainage from the stoma site. The urologist should be informed if there is significant leakage around a suprapubic catheter. Some leakage is expected, but larger amounts should be communicated to the care team. Cleansing the skin is appropriate but does not resolve the problem. Removing the suprapubic tube is contraindicated because it is unsafe. Administering drugs will not stop the leakage of urine around the tube.

A client who is scheduled for an open prostatectomy is concerned about the potential effects of the surgery on sexual function. What aspect of prostate surgery should inform the nurse's response? A. Erectile dysfunction is common after prostatectomy as a result of hormonal changes. B. All prostatectomies carry a risk of nerve damage and consequent erectile dysfunction. C. Erectile dysfunction after prostatectomy is expected, but normally resolves within several months. D. Modern surgical techniques have eliminated the risk of erectile dysfunction following prostatectomy.

Ans: B Rationale: All prostatectomies carry a risk of impotence because of potential damage to the pudendal nerves. If this damage occurs, the effects are permanent. Hormonal changes do not affect sexual functioning after prostatectomy.

The nurse is providing care for a client who has just been diagnosed with peripheral arterial occlusive disease (PAD). What assessment finding is most consistent with this diagnosis? A. Numbness and tingling in the distal extremities B. Unequal peripheral pulses between extremities C. Visible clubbing of the fingers and toes D. Reddened extremities with muscle atrophy

Ans: B Rationale: PAD assessment may manifest as unequal pulses between extremities, with the affected leg cooler and paler than the unaffected leg. Intermittent claudication is far more common than sensations of numbness and tingling. Clubbing and muscle atrophy are not associated with PAD.

A 20-year-old client with no medical history arrives at a walk-in/urgent care clinic reporting swelling on the left side of the neck. On palpation, the lymph nodes on the neck are painless, firm but not hard. What is the next appropriate intervention for this client? A. Recommend immediate and urgent transfer to the nearest trauma center. B. Perform diagnostic studies to rule out any infectious origin at a hospital. C. Refer the client to a primary health care provider for a nonurgent appointment. D. Complete a computed tomography scan because the client has Hodgkin lymphoma.

Ans: B Rationale: Although a high suspicion of Hodgkin lymphoma is present, diagnosis is premature prior to ruling out any infectious origin with diagnostic testing. This testing is by excisional node biopsy and usually done at a surgical center or hospital. Transfer is not an urgent manner unless the swelling is impacting the airway. Hodgkin lymphoma usually begins as an enlargement of one or more lymph nodes on one side of the neck. The individual nodes are painless and firm but not hard. It is also more common in males with 2 peaks in age groups. The first peak is between 15-34 and the second is after 60 years of age. Because these findings are consistent with Hodgkin lymphoma, a hospital admission, not a nonurgent appointment, is appropriate. Chest x-ray, computed tomography scan, and positron emission tomography scan are all involved in staging of Hodgkin lymphoma.

A nurse in a long-term care facility is caring for an 83-year-old client who has a history of heart failure (HF) and peripheral arterial disease (PAD). At present, the client is unable to stand or ambulate. The nurse should implement measures to prevent which complication? A. Aortitis B. Deep vein thrombosis C. Thoracic aortic aneurysm D. Raynaud disease

Ans: B Rationale: Although the exact cause of venous thrombosis remains unclear, three factors, known as Virchow triad, are believed to play a significant role in its development: stasis of blood (venous stasis), vessel wall injury, and altered blood coagulation. This client has venous stasis from immobility, vessel wall injury from PAD, and altered blood coagulation from HF. The cause of aortitis is unknown, but it has no direct connection to HF, PAD, or mobility issues. The greatest risk factors for thoracic a

A client is being treated in the ICU after a medical error resulted in an acute hemolytic transfusion reaction. What was the etiology of this client's adverse reaction? A. Antibodies to donor leukocytes remained in the blood. B. The donor blood was incompatible with that of the client. C. The client had a sensitivity reaction to a plasma protein in the blood. D. The blood was infused too quickly and overwhelmed the client's circulatory system.

Ans: B Rationale: An acute hemolytic reaction occurs when the donor blood is incompatible with that of the recipient. In the case of a febrile nonhemolytic reaction, antibodies to donor leukocytes remain in the unit of blood or blood component. An allergic reaction is a sensitivity reaction to a plasma protein within the blood component. Hypervolemia does not cause an acute hemolytic reaction.

A client is undergoing diagnostic testing for chronic lymphocytic leukemia (CLL). Which assessment finding is certain to be present if the client has CLL? A. Increased numbers of blast cells B. Increased lymphocyte levels C. Intractable bone pain D. Thrombocytopenia with no evidence of bleeding

Ans: B Rationale: An increased lymphocyte count (lymphocytosis) is always present in clients with CLL. Each of the other listed symptoms may or may not be present, and none is definitive for CLL.

A client's electronic health record notes that the client has previously undergone treatment for secondary polycythemia. The nurse should assess for which factor? A. Recent blood donation B. Evidence of lung disease C. A history of venous thromboembolism D. Impaired renal function

Ans: B Rationale: Any reduction in oxygenation, such as lung disease, can cause secondary polycythemia. Blood donation does not precipitate this problem and impaired renal function typically causes anemia, not polycythemia. A history of venous thromboembolism is not a likely contributor.

The staff educator is presenting a workshop on valvular disorders. When discussing the pathophysiology of aortic regurgitation, the educator describes what consequence of this disorder? A. Cardiac tamponade B. Left ventricular hypertrophy C. Right-sided heart failure D. Ventricular insufficiency

Ans: B Rationale: Aortic regurgitation eventually causes left ventricular hypertrophy. In aortic regurgitation, blood from the aorta returns to the left ventricle during diastole in addition to the blood normally delivered by the left atrium. The left ventricle dilates, trying to accommodate the increased volume of blood. Aortic regurgitation does not cause cardiac tamponade, right-sided heart failure, or ventricular insufficiency.

A client with a new diagnosis of leukemia is about to start treatment and expresses fear and anxiety with the prognosis. Which action is the nurse's most appropriate? A. Communicate to the healthcare provider the need to provide more information to the client and family. B. Assess how much information is desired from the client in terms of illness, treatment, and complications. C. Offer to call pastoral services and review hospice and/or palliative care so the client can have a quiet, dignified death. D. Encourage the client to call their family and discuss immediate role restructuring in both their family and professional life.

Ans: B Rationale: As with any client exhibiting anxiety and fear about a prognosis, listening should come first in order to assess how much information the client wants to have regarding the illness, treatment, and potential complications. This is an ongoing assessment, since needs and interest in information changes throughout the course of treatment. Managing a client's care is a team effort, so involving the primary care provider and family is important, but not the nurse's priority action. Offering pastoral services and role restructuring has its place in treatment but should be discussed after an assessment of the client's needs. A discussion about palliative care and hospice is not appropriate at this time. Offering realistic hope is important, and only after all treatment options are exhausted, or the client is diagnosed as terminal, should palliative and/or hospice care be considered.

A nurse is preparing health education for a client who has received a diagnosis of myelodysplastic syndrome (MDS). Which of the following topics should the nurse prioritize? A. Techniques for energy conservation and activity management B. Emergency management of bleeding episodes C. Technique for the administration of bronchodilators by metered-dose inhaler D. Techniques for self-palpation of the lymph nodes

Ans: B Rationale: Because of clients' risks of hemorrhage, clients with MDS should be taught techniques for managing emergent bleeding episodes. Bronchodilators are not indicated for the treatment of MDS and lymphedema is not normally associated with the disease. Energy conservation techniques are likely to be useful, but management of hemorrhage is a priority because of the potential consequences.

A client on the medical unit is receiving a unit of packed red blood cells (PRBCs). Difficult intravenous (IV) access has necessitated a slow infusion rate, and the nurse notes that the infusion began 4 hours ago. Which action by the nurse is the most appropriate? A. Apply an ice pack to the blood that remains to be infused. B. Discontinue the remainder of the PRBC transfusion, and inform the health care provider. C. Disconnect the bag of PRBCs, cool for 30 minutes, and then administer. D. Administer the remaining PRBCs by the IV direct (IV push) route.

Ans: B Rationale: Because of the risk of increased bacterial proliferation in the PRBCs and subsequent infection in the client, a PRBC transfusion should not exceed 4 hours. Remaining blood should not be transfused, even if it is cooled. Blood is not administered by the IV direct route.

A client is being treated for the effects of a longstanding vitamin B12 deficiency. Which aspect of the client's health history would most likely predispose the client to this deficiency? A. The client has irregular menstrual periods. B. The client is a vegan. C. The client donated blood 60 days ago. D. The client frequently smokes marijuana.

Ans: B Rationale: Because vitamin B12 is found only in foods of animal origin, vegans may ingest little vitamin B12. Irregular menstrual periods, marijuana use, and blood donation would not precipitate a vitamin B12 deficiency.

A client presents to the emergency department with paraphimosis. The health care provider is able to compress the glans and manually reduce the edema. Once the inflammation and edema subside, the nurse should prepare the client for what intervention? A. Needle aspiration of the corpus cavernosum B. Circumcision C. Abstinence from sexual activity for 6 weeks D. Administration of vardenafil

Ans: B Rationale: Circumcision is usually indicated after the inflammation and edema subside. Needle aspiration of the corpus cavernosum is indicated in priapism; abstinence from sexual activity for 6 weeks is not indicated. Vardenafil is used for erectile dysfunction and would not be used for paraphimosis.

A client is scheduled for a splenectomy. During discharge education, which teaching point should the nurse prioritize? A. Adhering to prescribed immunosuppressant therapy B. Reporting any signs or symptoms of infection promptly C. Ensuring adequate folate, iron, and vitamin B12 intake D. Limiting activity postoperatively to prevent hemorrhage

Ans: B Rationale: Clients face an increased risk for infection following splenectomy; therefore, long-term use of antibiotic therapy is indicated. After splenectomy, the client is instructed to seek prompt medical attention if even relatively minor symptoms of infection occur. Often, clients with high platelet counts have even higher counts after splenectomy, which can predispose them to serious thrombotic or hemorrhagic problems. However, this increase is usually transient and therefore often does not warrant additional treatment. Dietary modifications are not normally necessary, and immunosuppressants would be strongly contraindicated.

A nurse is providing discharge education to a client who has recently been diagnosed with a bleeding disorder. Which topic should the nurse prioritize when teaching this client? A. Avoiding buses, subways, and other crowded, public sites B. Avoiding activities that carry a risk for injury C. Keeping immunizations current D. Avoiding foods high in vitamin K

Ans: B Rationale: Clients with bleeding disorders need to understand the importance of avoiding activities that increase the risk of bleeding, such as contact sports. Immunizations involve injections and may be contraindicated for some clients. Clients with bleeding disorders do not need to normally avoid crowds. Foods high in vitamin K may be beneficial, not detrimental.

A 55-year-old client with a history of type 1 diabetes presents at the clinic reporting erectile dysfunction. The health care provider prescribes tadalafil to be taken 1 hour before sexual intercourse. The nurse reviews the client's history prior to instructing the client on the use of this medication. What disorder will contraindicate the use of tadalafil? A. Cataracts B. Retinopathy C. Diabetic neuropathy D. Diabetic nephropathy

Ans: B Rationale: Clients with cataracts, hypotension, or nephropathy will be allowed to take tadalafil and sildenafil, if needed. However, they are usually contraindicated with diabetic retinopathy.

A client with mitral valve stenosis is receiving health education at an outpatient clinic. To minimize the client's symptoms, the nurse would teach the client to take which action? A. Eat a high-protein, low-carbohydrate diet. B. Avoid activities that cause an increased heart rate. C. Avoid large crowds and public events. D. Perform deep breathing and coughing exercises.

Ans: B Rationale: Clients with mitral stenosis are advised to avoid strenuous activities, competitive sports, and pregnancy, all of which increase heart rate. Infection prevention is important, but avoiding crowds is not usually necessary. Deep breathing and coughing are not likely to prevent exacerbations of symptoms and increased protein intake is not necessary.

A client diagnosed with acute myeloid leukemia has just been admitted to the oncology unit. When writing this client's care plan, which potential complication should the nurse address? A. Pancreatitis B. Hemorrhage C. Arteritis D. Liver dysfunction

Ans: B Rationale: Complications of AML include bleeding and infection, which are the major causes of death. The risk of bleeding correlates with the level and duration of platelet deficiency. The low platelet count can cause ecchymoses and petechiae. Major hemorrhages also may develop when the platelet count drops to less than 10,000/mm3. The most common bleeding sources include gastrointestinal (GI), pulmonary, vaginal, and intracranial. Pancreatitis, arteritis, and liver dysfunction are generally not complications of leukemia.

A health care provider has explained to the client that they have an inflammation of the Cowper glands. Where are the Cowper glands located? A. Within the epididymis B. Below the prostate, within the posterior aspect of the urethra C. On the inner epithelium lining the scrotum, lateral to the testes D. Medial to the vas deferens

Ans: B Rationale: Cowper glands lie below the prostate, within the posterior aspect of the urethra. This gland empties its secretions into the urethra during ejaculation, providing lubrication. The Cowper glands do not lie within the epididymis, within the scrotum, or alongside the vas deferens.

The nurse is reviewing the echocardiography results of a client who has just been diagnosed with dilated cardiomyopathy (DCM). What changes in heart structure is this client experiencing? A. Dilated ventricles with atrophy of the ventricles B. Dilated ventricles without hypertrophy of the ventricles C. Dilation and hypertrophy of all four heart chambers D. Dilation of the atria and hypertrophy of the ventricles

Ans: B Rationale: DCM is characterized by significant dilation of the ventricles without significant concomitant hypertrophy and systolic dysfunction. The ventricles do not atrophy in clients with DCM.

The nurse is providing care for a 73-year-old client who has a hematologic disorder. Which change in hematologic function is age-related? A. Bone marrow in older adults produces a smaller proportion of healthy, functional blood cells. B. Older adults are less able to increase blood cell production when demand suddenly increases. C. Stem cells in older adults eventually lose their ability to differentiate. D. The ratio of plasma to erythrocytes and lymphocytes increases with age.

Ans: B Rationale: Due to a variety of factors, when an older person needs more blood cells, the bone marrow may not be able to increase production of these cells adequately. Stem cell activity continues throughout the lifespan, although at a somewhat decreased rate. The proportion of functional cells does not greatly decrease, and the relative volume of plasma does not change significantly.

A client with a documented history of glucose-6-phosphate dehydrogenase deficiency has presented to the emergency department with signs and symptoms including pallor, jaundice, and malaise. Which of the nurse's assessment questions relates most directly to this client's hematologic disorder? A. "When did you last have a blood transfusion?" B. "What medications have you taken recently?" C. "Have you been under significant stress lately?" D. "Have you suffered any recent injuries?"

Ans: B Rationale: Exacerbations of glucose-6-phosphate dehydrogenase deficiency are nearly always precipitated by medications. Blood transfusions, stress, and injury are less common triggers.

A client has sustained a cut to the hand, immediately initiating the process of hemostasis. Following vasoconstriction, which event in the process of hemostasis will take place? A. Fibrin will be activated at the bleeding site. B. Platelets will aggregate at the injury site. C. Thromboplastin will form a clot. D. Prothrombin will be converted to thrombin.

Ans: B Rationale: Following vasoconstriction, circulating platelets aggregate at the site and adhere to the vessel and to one another, forming an unstable hemostatic plug. Events involved in the clotting cascade take place subsequent to this initial platelet action.

A client's health history reveals daily consumption of two to three bottles of wine. The nurse would consider increased risk for which hematologic disorder when planning assessments and interventions for this client? A. Leukemia B. Anemia C. Thrombocytopenia D. Lymphoma

Ans: B Rationale: Heavy alcohol use is associated with numerous health problems, including anemia. Leukemia and lymphoma are not associated with alcohol use; red blood cell levels are typically affected more than platelet levels (i.e., thrombocytopenia).

A client is being discharged home after a heart transplant. The nurse is preparing to provide medication education on cyclosporine and tacrolimus. Which nursing diagnosis would be prioritized for this client? A. Risk for injury B. Risk for infection C. Risk for peripheral neurovascular dysfunction D. Risk for unstable blood glucose

Ans: B Rationale: Immunosuppressants decrease the body's ability to resist infections, and a satisfactory balance must be achieved between suppressing rejection and avoiding infection. These drugs do not create a heightened risk of injury, neurovascular dysfunction, or unstable blood glucose levels.

The nurse is caring for a client with mitral stenosis who is scheduled for a balloon valvuloplasty. The client reports being unsure of why the surgeon did not opt to replace the damaged valve rather than repair it. Which statement indicates an advantage of valvuloplasty that the nurse should cite? A. The procedure can be performed on an outpatient basis in a healthcare provider's office. B. Repaired valves tend to function longer than replaced valves. C. The procedure is not associated with a risk of infection. D. Lower doses of antirejection drugs are required than with valve replacement.

Ans: B Rationale: In general, valves that undergo valvuloplasty function longer than prosthetic valve replacements and clients do not require continuous anticoagulation. Valvuloplasty carries a risk of infection, like all surgical procedures, and it is not performed in a health care provider's office. Antirejection drugs are unnecessary because foreign tissue is not introduced.

A nurse is caring for a client with acute myeloid leukemia who is preparing to undergo induction therapy. In preparing a plan of care for this client, the nurse should assign the highest priority to which nursing diagnosis? A. Activity intolerance B. Risk for infection C. Acute confusion D. Risk for spiritual distress

Ans: B Rationale: Induction therapy places the client at risk for infection, thus this is the priority nursing diagnosis. During the time of induction therapy, the client is very ill, with bacterial, fungal, and occasional viral infections; bleeding and severe mucositis, which causes diarrhea; and marked decline in the ability to maintain adequate nutrition. Supportive care consists of administering blood products and promptly treating infections. Immobility, confusion, and spiritual distress are possible, but infection is the client's most acute physiologic threat.

A night nurse is reviewing the next day's medication administration record (MAR) of a hospital client who has hemophilia. The nurse notes that the MAR specifies both oral and subcutaneous options for the administration of a PRN antiemetic. What is the nurse' best action? A. Ensure that the day nurse knows not to give the antiemetic. B. Contact the prescriber to have the subcutaneous option discontinued. C. Reassess the client's need for antiemetics. D. Remove the subcutaneous route from the client's MAR.

Ans: B Rationale: Injections must be avoided in clients with hemophilia. Consequently, the nurse should ensure that the prescriber makes the necessary change. The nurse cannot independently make a change to a client's MAR in most cases. Facilitating the necessary change is preferable to deferring to the day nurse.

A nurse practitioner is assessing a client who has a fever, malaise, and a white blood cell count that is elevated. What principle should guide the nurse's management of the client's care? A. There is a need for the client to be assessed for lymphoma. B. Infection is the most likely cause of the client's change in health status. C. The client is exhibiting signs and symptoms of leukemia. D. The client should undergo diagnostic testing for multiple myeloma.

Ans: B Rationale: Leukocytosis is most often the result of infection. It is only considered pathologic (and suggestive of leukemia) if it is persistent and extreme. Multiple myeloma and lymphoma are not likely causes of this constellation of symptoms.

The nurse is describing normal erythrocyte physiology to a client who has a diagnosis of anemia. The nurse should explain that the erythrocytes consist primarily of which substance? A. Plasminogen B. Hemoglobin C. Hematocrit D. Fibrin

Ans: B Rationale: Mature erythrocytes consist primarily of hemoglobin, which contains iron and makes up 95% of the cell mass. Erythrocytes are not made of fibrin or plasminogen. Hematocrit is a measure of erythrocyte volume in whole blood.

The nurse is caring for a client who is diagnosed with Raynaud phenomenon. The nurse should plan interventions to address which nursing diagnosis? A. Chronic pain B. Ineffective tissue perfusion C. Impaired skin integrity D. Risk for injury

Ans: B Rationale: Raynaud phenomenon is a form of intermittent arteriolar vasoconstriction resulting in inadequate tissue perfusion. This results in coldness, pain, and pallor of the fingertips or toes. Pain is typically intermittent and acute, not chronic, and skin integrity is rarely at risk. In most cases, the client is not at a high risk for injury.

A client has a diagnosis of multiple myeloma and the nurse is preparing health education in preparation for discharge from the hospital. Which action should the nurse promote? A. Daily performance of weight-bearing exercise to prevent muscle atrophy B. Close monitoring of urine output and kidney function C. Daily administration of warfarin, as prescribed D. Safe use of supplementary oxygen in the home setting

Ans: B Rationale: Renal function must be monitored closely in the client with multiple myeloma. Excessive weight-bearing can cause pathologic fractures. There is no direct indication for anticoagulation or supplementary oxygen.

A 35-year-old father of three tells the nurse that they want information on a vasectomy. What would the nurse tell the client about ejaculate after a vasectomy? A. There will be no ejaculate after a vasectomy, though the client's potential for orgasm is unaffected. B. There is no noticeable decrease in the amount of ejaculate even though it contains no sperm. C. There is a marked decrease in the amount of ejaculate after vasectomy, though this does not affect sexual satisfaction. D. There is no change in the quantity of ejaculate after vasectomy, but the viscosity is somewhat increased.

Ans: B Rationale: Seminal fluid is manufactured predominantly in the seminal vesicles and prostate gland, which are unaffected by vasectomy, thus no noticeable decrease in the amount of ejaculate occurs (volume decreases approximately 3%), even though it contains no spermatozoa. The viscosity of ejaculate does not change.

A client who is undergoing consolidation therapy for the treatment of leukemia has been experiencing debilitating fatigue. How can the nurse best meet this client's needs for physical activity? A. Teach the client about the risks of immobility and the benefits of exercise. B. Assist the client to a chair during awake times, as tolerated. C. Collaborate with the physical therapist to arrange for stair exercises. D. Teach the client to perform deep breathing and coughing exercises.

Ans: B Rationale: Sitting up in a chair is preferable to bed rest, even if a client is experiencing severe fatigue. A client who has debilitating fatigue would not likely be able to perform stair exercises. Teaching about mobility may be necessary, but education must be followed by interventions that actually involve mobility. Deep breathing and coughing reduce the risk of respiratory complications but are not substitutes for physical mobility in preventing deconditioning.

A client is receiving the first of two prescribed units of PRBCs. Shortly after the initiation of the transfusion, the client reports chills and experiences a sharp increase in temperature. What is the nurse's priority action? A. Position the client in high Fowler position B. Discontinue the transfusion. C. Auscultate the client's lungs. D. Obtain a blood specimen from the client.

Ans: B Rationale: Stopping the transfusion is the first step in any suspected transfusion reaction. This must precede other assessments and interventions, including repositioning, chest auscultation, and collecting specimens.

A client has been prescribed sildenafil (Viagra). What should the nurse teach the client about this medication? A. Sexual stimulation is not needed to obtain an erection. B. The drug should be taken 1 hour prior to intercourse. C. Facial flushing or headache should be reported to the health care provider. D. The drug has the potential to cause permanent visual changes.

Ans: B Rationale: The client must have sexual stimulation to create the erection, and the drug should be taken 1 hour before intercourse. Facial flushing, mild headache, indigestion, and running nose are common side effects of Viagra and do not normally warrant reporting. Some visual disturbances may occur, but these are transient.

A client with sickle cell disease is taking narcotic analgesics for pain control. Which intervention by the nurse would decrease the risk for narcotic substance abuse? A. Encourage the client to rely on complementary and alternative therapies. B. Encourage the client to seek care from a single provider for pain relief. C. Teach the client to accept chronic pain as an inevitable aspect of the disease. D. Limit the reporting of emergency department visits to the primary healthcare provider.

Ans: B Rationale: The client should be encouraged to use a single primary provider to address health care concerns. Emergency department visits should be reported to the primary provider to achieve optimal management of the disease. It would be inappropriate to teach the client to simply accept the pain. Complementary therapies are usually insufficient to fully address pain in sickle cell disease.

A nurse is caring for a client who has been diagnosed with leukemia. The nurse's most recent assessment reveals the presence of ecchymoses on the client's sacral area and petechiae on the forearms. In addition to informing the client's primary care provider, the nurse should perform what action? A. Initiate measures to prevent venous thromboembolism (VTE). B. Check the client's most recent platelet level. C. Place the client on protective isolation. D. Ambulate the client to promote circulatory function.

Ans: B Rationale: The client's signs are suggestive of thrombocytopenia, thus the nurse should check the client's most recent platelet level. VTE is not a risk and this does not constitute a need for isolation. Ambulation and activity may be contraindicated due to the risk of bleeding.

A client with acute kidney injury has decreased erythropoietin production. Upon analysis of the client's complete blood count, the nurse will expect which of the following results? A. An increased hemoglobin and decreased hematocrit. B. A decreased hemoglobin and hematocrit. C. A decreased mean corpuscular volume (MCV) and red cell distribution width (RDW). D. An increased mean corpuscular volume (MCV) and red cell distribution width (RDW).

Ans: B Rationale: The decreased production of erythropoietin will result in a decreased hemoglobin and hematocrit. The client will have normal MCV and RDW because the erythrocytes are normal in appearance.

A client newly admitted to the telemetry unit is experiencing progressive fatigue, hemoptysis, and dyspnea. Diagnostic testing has revealed that these signs and symptoms are attributable to pulmonary venous hypertension. Which valvular disorder would the nurse anticipate being diagnosed in this client? A. Aortic regurgitation B. Mitral stenosis C. Mitral valve prolapse D. Aortic stenosis

Ans: B Rationale: The first symptom of mitral stenosis is often dyspnea on exertion as a result of pulmonary venous hypertension. Symptoms usually develop after the valve opening is reduced by one-third to one-half its usual size. Clients are likely to show progressive fatigue as a result of low cardiac output. The enlarged left atrium may create pressure on the left bronchial tree, resulting in a dry cough or wheezing. Clients may expectorate blood (i.e., hemoptysis) or experience palpitations, orthopnea, par oxysmal nocturnal dyspnea, and repeated respiratory infections. Pulmonary venous hypertension is not typically caused by aortic regurgitation, mitral valve prolapse, or aortic stenosis.

A group of nurses are learning about the high incidence and prevalence of anemia among different populations. Which individual is most likely to have anemia? A. A 50-year-old black woman who is going through menopause B. An 81-year-old woman who has chronic heart failure C. A 48-year-old man who travels extensively and has a high-stress job D. A 13-year-old girl who has just experienced menarche

Ans: B Rationale: The incidence and prevalence of anemia are exceptionally high among older adults, and the risk of anemia is compounded by the presence of heart disease. None of the other listed individuals exhibits high-risk factors for anemia, though exceptionally heavy menstrual flow can result in anemia.

An adult client's abnormal complete blood count (FBC) and physical assessment have prompted the primary care provider to order a diagnostic workup for Hodgkin lymphoma. The presence of what assessment finding is considered diagnostic of the disease? A. Schwanncells B. Reed-Sternberg cells C. Lewy bodies D. LoopsofHenle

Ans: B Rationale: The malignant cell of Hodgkin lymphoma is the Reed-Sternberg cell, a gigantic tumor cell that is morphologically unique and thought to be of immature lymphoid origin. It is the pathologic hallmark and essential diagnostic criterion. Schwann cells exist in the peripheral nervous system and Lewy bodies are markers of Parkinson disease. Loops of Henle exist in nephrons.

A client with pericarditis has just been admitted to the critical care unit. The nurse planning the client's care should prioritize which nursing diagnosis? A. Anxiety related to pericarditis B. Acute pain related to pericarditis C. Ineffective tissue perfusion related to pericarditis D. Ineffective breathing pattern related to pericarditis

Ans: B Rationale: The most characteristic symptom of pericarditis is chest pain, although pain also may be located beneath the clavicle, in the neck, or in the left trapezius (scapula) region. The pain or discomfort usually remains fairly constant, but it may worsen with deep inspiration and when lying down or turning. Anxiety is highly plausible and should be addressed, but chest pain is a nearly certain accompaniment to the disease. Breathing and tissue perfusion are likely to be at risk, but pain is certain, especially in the early stages of treatment.

A nurse has written a plan of care for a client diagnosed with peripheral arterial insufficiency. One of the nursing diagnoses in the care plan is altered peripheral tissue perfusion related to compromised circulation. Which intervention is the most appropriate for this diagnosis? A. Elevate the legs and arms above the heart when resting. B. Encourage the client to engage in a moderate amount of exercise. C. Encourage extended periods of sitting or standing. D. Discourage walking in order to limit pain.

Ans: B Rationale: The nursing diagnosis of altered peripheral tissue perfusion related to compromised circulation requires interventions that focus on improving circulation. Encouraging the client to engage in a moderate amount of exercise serves to improve circulation. Elevating the client's legs and arms above the heart when resting would be passive and fails to promote circulation. Encouraging long periods of sitting or standing would further compromise circulation. The nurse should encourage, not discourage, walking to increase circulation and decrease pain.

A client undergoing a hip replacement has autologous blood on standby if a transfusion is needed. What is the primary advantage of autologous transfusions? A. Safe transfusion for clients with a history of transfusion reactions B. Prevention of viral infections from another person's blood C. Avoidance of complications in clients with alloantibodies D. Prevention of alloimmunization

Ans: B Rationale: The primary advantage of autologous transfusions is the prevention of viral infections from another person's blood. Other secondary advantages include safe transfusion for clients with a history of transfusion reactions, prevention of alloimmunization, and avoidance of complications in clients with alloantibodies.

A client newly diagnosed with thrombocytopenia is admitted to the medical unit. After the admission assessment, the client asks the nurse to explain the condition. The nurse explains to this client that this condition occurs due to which factor? A. An attack on the platelets by antibodies. B. Decreased production of platelets. C. Impaired communication between platelets. D. An autoimmune process causing platelet malfunction.

Ans: B Rationale: Thrombocytopenia can result from a decreased platelet production, increased platelet destruction, or increased consumption of platelets. Impaired platelet communication, antibodies, and autoimmune processes are not typical pathologies.

A client has been admitted to the medical unit with signs and symptoms suggestive of endocarditis. The health care provider's choice of antibiotics would be primarily based on what diagnostic test? A. Echocardiography B. Blood cultures C. Cardiac aspiration D. Full blood count

Ans: B Rationale: To help determine the causative organisms and the most effective antibiotic treatment for the client, blood cultures are taken. A CBC can help establish the degree and stage of infection, but not the causative microorganism. Echocardiography cannot indicate the microorganisms causing the infection. "Cardiac aspiration" is not a diagnostic test.

A postsurgical client has illuminated the call light to inform the nurse of a sudden onset of lower leg pain. On inspection, the nurse observes that the client's left leg is visibly swollen and reddened. Which action by the nurse would be most appropriate? A. Administer a PRN dose of subcutaneous heparin. B. Inform the health care provider that the client has signs and symptoms of venous thromboembolism (VTE). C. Mobilize the client promptly to dislodge any thrombi in the client's lower leg. D. Massage the client's lower leg to temporarily restore venous return.

Ans: B Rationale: VTE requires prompt medical follow-up. Heparin will not dissolve an established clot. Massaging the client's leg and mobilizing the client would be contraindicated because they would dislodge the clot, possibly resulting in a pulmonary embolism.

The nurse caring for a client receiving a transfusion notes that 15 minutes after the infusion of packed red blood cells (PRBCs) has begun, the client is having difficulty breathing and reports severe chest tightness. What is the most appropriate initial action for the nurse to take? A. Notify the client's healthcare provider. B. Stop the transfusion immediately. C. Remove the client's IV access. D. Assess the client's chest sounds and vital signs.

Ans: B Rationale: Vascular collapse, bronchospasm, laryngeal edema, shock, fever, chills, and jugular vein distension are severe reactions. The nurse should discontinue the transfusion immediately, monitor the client's vital signs, and notify the health care provider. The blood container and tubing should be sent to the blood bank. A blood and urine specimen may be needed if a transfusion reaction or a bacterial infection is suspected. The client's IV access should not be removed.

A client's blood work reveals a platelet level of 17,000/mm3. When inspecting the client's integumentary system, what finding would be most consistent with this platelet level? A. Dermatitis. B. Petechiae. C. Urticaria. D. Alopecia.

Ans: B Rationale: When the platelet count drops to less than 20,000/mm3, petechiae can appear. Low platelet levels do not normally result in dermatitis, urticaria (hives), or alopecia (hair loss).

A clinic nurse is providing preprocedure education for a man who will undergo a vasectomy. Which of the following measures will enhance healing and comfort? Select all that apply. A. Abstaining from sexual intercourse for at least 14 days post-procedure B. Wearing a scrotal support garment C. Using sitz baths D. Applying a heating pad intermittently E. Staying on bed rest for 48 to 72 hours post-procedure

Ans: B, C Rationale: Applying ice bags intermittently to the scrotum for several hours after surgery can reduce swelling and relieve discomfort, and is preferable to the application of heat. The nurse advises the client to wear snug, cotton underwear or a scrotal support for added comfort and support. Sitz baths can also enhance comfort. Extended bed rest is unnecessary, and sexual activity can usually be resumed in 1 week.

A man comes to the clinic reporting difficulty obtaining an erection. When reviewing the client's history, what might the nurse note that contributes to erectile dysfunction? Select all that apply. A. The client has been treated for a UTI twice in the past year. B. The client has a history of hypertension. C. The client is 66 years old. D. The client leads a sedentary lifestyle. E. The client drinks five to six alcoholic drinks per day.

Ans: B, E Rationale: Past history of infection and lack of exercise do not contribute to impotence. With advancing age, sexual function and libido and potency decrease somewhat, but this is not the primary reason for impotence. Vascular problems cause about half the cases of impotence in men older than 50 years; hypertension is a major cause of such problems. Heavy alcohol use can contribute to the problem as well.

A young man with a diagnosis of hemophilia A has been brought to emergency department after suffering a workplace accident resulting in bleeding. Rapid assessment has revealed the source of the client's bleeding and established that his vital signs are stable. What should be the nurse's next action? A. Position the client in a prone position to minimize bleeding. B. Establish IV access for the administration of vitamin K. C. Prepare for the administration of factor VIII. D. Administer a normal saline bolus to increase circulatory volume.

Ans: C Rationale: Injuries to clients with hemophilia necessitate prompt administration of clotting factors. Vitamin K is not a treatment modality and a prone position will not be appropriate for all types and locations of wounds. A normal saline bolus is not indicated.

A nurse in a dermatology clinic is reading the electronic health record of a new client. The nurse notes that the client has a history of a primary skin lesion. What skin lesion may this client have? A. Crust B. Keloid C. Pustule D. Ulcer

Ans: C Rationale: A pustule is an example of a primary skin lesion. Primary skin lesions are original lesions arising from previously normal skin. Crusts, keloids, and ulcers are secondary lesions.

A new client has come to the dermatology clinic to be assessed for a reddened rash on the abdomen. For what diagnostic test should the nurse prepare the client to identify the causative allergen? A. Skinscrapings B. Skin biopsy C. Patch testing D. Tzancksmear

Ans: C Rationale: Patch testing is performed to identify substances to which the client has developed an allergy. Skin scrapings are done for suspected fungal lesions. A skin biopsy is completed to rule out malignancy and to establish an exact diagnosis of skin lesions. A Tzanck smear is used to examine cells from blistering skin conditions, such as herpes zoster.

While assessing a 25-year-old female, the nurse notes that the client has hair on her lower abdomen. Earlier in the health interview, the client stated that her menses are irregular. The nurse should suspect what type of health problem? A. A metabolic disorder B. A malignancy C. A hormonal imbalance D. An infectious process

Ans: C Rationale: Some women with higher levels of testosterone have hair in the areas generally thought of as masculine, such as the face, chest, and lower abdomen. This is often a normal genetic variation, but if it appears along with irregular menses and weight changes, it may indicate a hormonal imbalance. This combination of irregular menses and hair distribution is inconsistent with metabolic disorders, malignancy, or infection.

When planning the skin care of a client with decreased mobility, the nurse is aware of the varying thickness of the epidermis. At what location is the epidermal layer thickest? A. Thescalp B. The elbows C. The palms of the hands D. Theknees

Ans: C Rationale: The epidermis is the thickest over the palms of the hands and the soles of the feet.

An adolescent is identified as having a collection of fluid in the tunica vaginalis of their testes. The nurse knows that this adolescent will receive what medical diagnosis? A. Cryptorchidism B. Orchitis C. Hydrocele D. Prostatism

Ans: C Rationale: A hydrocele refers to a collection of fluid in the tunica vaginalis of the testis. Cryptorchidism is the most common congenital defect in males, characterized by failure of one or both of the testes to descend into the scrotum. Orchitis is an inflammation of the testes (testicular congestion) caused by pyogenic, viral, spirochetal, parasitic, traumatic, chemical, or unknown factors. Prostatism is an obstructive and irritative symptom complex that includes increased frequency and hesitancy in starting urination, a decrease in the volume and force of the urinary stream, acute urinary retention, and recurrent urinary tract infections.

The nurse is assessing a new client with reports of acute fatigue and a sore tongue that is visibly smooth and beefy red. This client is demonstrating signs and symptoms associated with what form of hematologic disorder? A. Sickle cell disease. B. Hemophilia. C. Megaloblastic anemia. D. Thrombocytopenia.

Ans: C Rationale: A red, smooth, sore tongue is a symptom associated with megaloblastic anemia. Sickle cell disease, hemophilia, and thrombocytopenia do not have symptoms involving the tongue.

A 79-year-old client is admitted to the medical unit with digital gangrene. The client reports that the problem first began when the client stubbed the toe going to the bathroom in the dark. In addition to this trauma, the nurse should suspect that the client has a history of which health problem? A. Raynaud phenomenon B. Coronary artery disease (CAD) C. Arterial insufficiency D. Varicose veins

Ans: C Rationale: Arterial insufficiency may result in gangrene of the toe (digital gangrene), which usually is caused by trauma. The toe is stubbed and then turns black. Raynaud disease, CAD, and varicose veins are not the usual causes of digital gangrene in older adults.

A client with several chronic health problems has been newly diagnosed with a qualitative platelet defect. What component of the client's previous medication regimen may have contributed to the development of this disorder? A. Calcium carbonate B. Vitamin B12 C. Aspirin D. Vitamin D

Ans: C Rationale: Aspirin may induce a platelet disorder. Even small amounts of aspirin reduce normal platelet aggregation, and the prolonged bleeding time lasts for several days after aspirin ingestion. Calcium, vitamin D, and vitamin B12 do not have the potential to induce a platelet defect.

Which of the following circumstances would most clearly warrant autologous blood donation? A. The client has type-O blood. B. The client has sickle cell disease or a thalassemia. C. The client has elective surgery pending. D. The client has hepatitis C.

Ans: C Rationale: Autologous blood donation is useful for many elective surgeries where the potential need for transfusion is high. Type-O blood, hepatitis, sickle cell disease, and thalassemia are not clear indications for autologous donation.

An emergency department nurse is triaging a 77-year-old client who presents with uncharacteristic fatigue as well as back and rib pain. The client denies any recent injuries. The nurse should recognize the need for this client to be assessed for which health problem? A. Hodgkin disease B. Non-Hodgkin lymphoma C. Multiple myeloma D. Acute thrombocythemia

Ans: C Rationale: Back pain, which is often a presenting symptom in multiple myeloma, should be closely investigated in older clients. The lymphomas and bleeding disorders do not typically present with the primary symptom of back pain or rib pain.

A client with myelodysplastic syndrome (MDS) is being treated on a medical unit. Which priority finding should prompt the nurse to contact the client's primary care provider? A. Reports of a frontal lobe headache B. An episode of urinary incontinence C. An oral temperature of 37.5°C (99.5°F) D. An oxygen saturation (SpO2) of 91% on room air

Ans: C Rationale: Because the client with MDS is at a high risk for infection, any early signs of infection must be reported promptly. The nurse should address each of the listed assessment findings, but none is as direct a threat to the client's immediate health as an infection.

A nurse is providing an educational event to a local men's group about prostate cancer. The nurse should cite an increased risk of prostate cancer in what ethnic group? A. Native Americans/First Nations B. White C. Black D. Asian

Ans: C Rationale: Black men have the highest high risk of prostate cancer.

A client with a hematologic disorder asks the nurse how the body forms blood cells. The nurse understands that this process takes place primarily in which location? A. Spleen B. Kidneys C. Bone marrow D. Liver

Ans: C Rationale: Bone marrow is the primary site for hematopoiesis. The liver and spleen may be involved during embryonic development or when marrow is destroyed. The kidneys release erythropoietin, which stimulates the marrow to increase production of red blood cells. However, blood cells are not primarily formed in the spleen, kidneys, or liver.

The triage nurse in the emergency department is assessing a client who reports pain and swelling in the right lower leg. The client's pain became much worse last night and appeared along with fever, chills, and sweating. The client states, "I hit my leg on the car door 4 or 5 days ago, and it has been sore ever since." The client has a history of chronic venous insufficiency. Which intervention should the nurse anticipate for this client? A. Platelet transfusion to treat thrombocytopenia B. Warfarin to treat arterial insufficiency C. Antibiotics to treat cellulitis D. Intravenous heparin to treat venous thromboembolism (VTE)

Ans: C Rationale: Cellulitis is the most common infectious cause of limb swelling. The signs and symptoms include acute onset of swelling, localized redness, and pain; it is frequently associated with systemic signs of fever, chills, and sweating. The client may be able to identify a trauma that accounts for the source of infection. Thrombocytopenia is a loss or decrease in platelets and increases a client's risk of bleeding; this problem would not cause these symptoms. Arterial insufficiency would present with ongoing pain related to activity. This client does not have signs and symptoms of VTE.

An older adult client has been treated for a venous ulcer and a plan is in place to prevent the occurrence of future ulcers. What should the nurse include in this plan? A. Use of supplementary oxygen to aid tissue oxygenation B. Daily use of normal saline compresses on the lower limbs C. Daily administration of prophylactic antibiotics D. A high-protein diet that is rich in vitamins

Ans: D Rationale: A diet that is high in protein, vitamins C and A, iron, and zinc is encouraged to promote healing and prevent future ulcers. Prophylactic antibiotics and saline compresses are not used to prevent ulcers. Oxygen supplementation does not prevent ulcer formation.

Fresh-frozen plasma (FFP) has been prescribed for a hospital client. Prior to administration of this blood product, the nurse should prioritize which client education? A. Infection risks associated with FFP administration B. Physiologic functions of plasma C. Signs and symptoms of a transfusion reaction D. Strategies for managing transfusion-associated anxiety

Ans: C Rationale: Clients should be educated about signs and symptoms of transfusion reactions prior to administration of any blood product. In most cases, this is priority over education relating to infection. Anxiety may be an issue for some clients, but transfusion reactions are life-threatening and should be addressed first. Teaching about the functions of plasma is not likely a high priority.

A client has been scheduled for a bone marrow aspiration and admits to the nurse being worried about the pain involved with the procedure. Which statement by the nurse when providing client education would be most accurate? A. "You'll be given painkillers before the test, so there won't likely be any pain." B. "You'll feel some pain when the needle enters your skin, but none during the aspiration." C. "Most people feel some brief, sharp pain when the marrow is aspirated." D. "I'll be there with you, and I'll try to help you keep your mind off the pain."

Ans: C Rationale: Clients typically feel a pressure sensation as the needle is advanced into position. The actual aspiration always causes sharp, brief pain, resulting from the suction exerted as the marrow is aspirated into the syringe; the client should be warned about this. Stating, "I'll try to help you keep your mind off the pain" may increase the client's fears of pain, because this does not help the client know what to expect. Although a local anesthetic agent is administered to the skin, subcutaneous tissue, and periosteum of the bone, it is not possible to anesthetize the bone itself, and the client will most likely experience sharp, brief pain during the actual aspiration. Painkillers are not necessarily given before the test and would not likely block all pain from the aspiration.

A nurse is planning the care of a client who has been admitted to the medical unit with a diagnosis of multiple myeloma. In the client's care plan, the nurse has identified a diagnosis of Risk for Injury, which should be attributed to which factor? A. Labyrinthitis B. Left ventricular hypertrophy C. Decreased bone density D. Hypercoagulation

Ans: C Rationale: Clients with multiple myeloma are at risk for pathologic bone fractures secondary to diffuse osteoporosis (decreased bone density) and osteolytic lesions. Labyrinthitis is uncharacteristic, and clients do not normally experience hypercoagulation or cardiac hypertrophy as a result of multiple myeloma.

A nurse is providing care to a client with multiple myeloma with reports of nausea, diarrhea, alopecia, and red urine. The client's recent interventions include electrocardiogram (ECG), multigated acquisition scan (MUGA), and a central line venous access placed on the right chest wall. Which medication is the client most likely receiving? A. Dexamethasone B. Lenalidomide C. Doxorubicin D. Etoposide

Ans: C Rationale: Doxorubicin is a chemotherapeutic drug and typically part of a combination regimen. Side effects of this medication include nausea, vomiting, alopecia (hair loss), and orange or red urine. Red urine is not listed as a side effect on any other of the medications listed. Doxorubicin can have a cardiotoxic effect (cardiomyopathy & arrhythmias), so ECG and MUGA scans (evaluate pumping function of the ventricles) are done before and periodically throughout treatment. This drug is a vesicant (causes blistering) and can result in tissue necrosis if the medication leaks into the tissues surrounding a vein due to an infiltrate. A central line is placed to avoid that complication. Etoposide and lenalidomide are both chemotherapy drugs without the typical adverse effect of cardiotoxicity. Etoposide is an irritant with a low vesicant potential. Dexamethasone is a steroid that does not have the side effect of alopecia nor requires central venous access.

A client who has undergone a valve replacement with a mechanical valve prosthesis is due to be discharged home. During discharge teaching, the nurse would discuss the importance of antibiotic prophylaxis prior to which event? A. Exposure to immunocompromised individuals B. Future hospital admissions C. Dental procedures D. Live vaccinations

Ans: C Rationale: Following mechanical valve replacement, antibiotic prophylaxis is necessary before dental procedures involving manipulation of gingival tissue, the periapical area of the teeth or perforation of the oral mucosa (not including routine anesthetic injections, placement of orthodontic brackets, or loss of deciduous teeth). There are no current recommendations around antibiotic prophylaxis prior to vaccination, future hospital admissions, or exposure to people who are immunosuppressed.

A woman who is in her third trimester of pregnancy has been experiencing an exacerbation of iron-deficiency anemia in recent weeks. When providing the client with nutritional guidelines and meal suggestions, what foods would be most likely to increase the woman's iron stores? A. Salmon accompanied by whole milk B. Mixed vegetables and brown rice C. Beef liver accompanied by orange juice D. Yogurt, almonds, and whole grain oats

Ans: C Rationale: Food sources high in iron include organ meats, other meats, beans (e.g., black and pinto), leafy green vegetables, raisins, and molasses. Taking iron-rich foods with a source of vitamin C (e.g., orange juice) enhances the absorption of iron. All of the listed foods are nutritious, but liver and orange juice are most likely to be of benefit to iron stores.

A nurse is caring for a client who is being treated for leukemia in the hospital. The client was able to maintain nutritional status for the first few weeks following the diagnosis but is now exhibiting early signs and symptoms of malnutrition. In collaboration with the dietitian, the nurse should implement what intervention? A. Arrange for total parenteral nutrition (TPN). B. Facilitate placement of a percutaneous endoscopic gastrostomy (PEG) tube. C. Provide the client with several small, soft-textured meals each day. D. Assign responsibility for the client's nutrition to the client's friends and family.

Ans: C Rationale: For clients experiencing difficulties with oral intake, the provision of small, easily chewed meals may be beneficial. This option would be trialed before resorting to tube feeding or TPN. The family should be encouraged to participate in care, but should not be assigned full responsibility.

A nurse at a long-term care facility is amending the care plan of a resident who has just been diagnosed with essential thrombocythemia (ET). The nurse should anticipate the administration of which medication? A. Dalteparin B. Allopurinol C. Hydroxyurea D. Hydrochlorothiazide

Ans: C Rationale: Hydroxyurea is effective in lowering the platelet count for clients with ET. Dalteparin, allopurinol, and hydrochlorothiazide do not have this therapeutic effect.

The nurse is caring for an acutely ill client who is on a factor Xa inhibitor. The client has a comorbidity of renal insufficiency. How will this client's renal status affect this anticoagulant therapy? A. The factor Xa inhibitor is contraindicated in the treatment of this client. B. The factor Xa inhibitor may be given subcutaneously, but not intravenously (IV). C. Lower doses of factor Xa inhibitor are required for this client. D. Warfarin will be substituted for the factor Xa inhibitor.

Ans: C Rationale: If renal insufficiency exists, lower doses, not contraindication, of factor Xa inhibitors are needed. Warfarin is not an acceptable substitution for this type of medication. There is no contraindication for IV administration.

A client with mitral stenosis exhibits new symptoms of a dysrhythmia. Based on the pathophysiology of this disease process, the nurse would expect the client to exhibit which heart rhythm? A. Ventricular fibrillation (VF) B. Ventricular tachycardia (VT) C. Atrial fibrillation D. Sinus bradycardia

Ans: C Rationale: In clients with mitral valve stenosis, the pulse is weak and often irregular because of atrial fibrillation caused by strain on the atrium. Bradycardia, VF, and VT are not characteristic of this valvular disorder.

After receiving a diagnosis of acute lymphocytic leukemia, a client is visibly distraught, stating, "I have no idea where to go from here." How should the nurse prepare to meet this client's psychosocial needs? A. Assess the client's previous experience with the healthcare system. B. Reassure the client that treatment will be challenging but successful. C. Assess the client's specific needs for education and support. D. Identify the client's plan of medical care.

Ans: C Rationale: In order to meet the client's needs, the nurse must first identify the specific nature of these needs. According to the nursing process, assessment must precede interventions. The plan of medical care is important, but not central to the provision of support. The client's previous health care is not a primary consideration, and the nurse cannot assure the client of successful treatment.

A nurse on a medical unit is caring for a client who has been diagnosed with lymphangitis. When reviewing this client's medication administration record, the nurse should anticipate which type of medication? A. An anticoagulant B. A diuretic C. An antibiotic D. An antiplatelet aggregator

Ans: C Rationale: Lymphangitis is an acute inflammation of the lymphatic channels caused by an infectious process. Antibiotics are always a component of treatment. Diuretics are of nominal use. Anticoagulants and antiplatelet aggregators are not indicated in this form of infection.

The nurse on the pediatric unit is caring for a 10-year-old child with a diagnosis of hemophilia. The nurse should assess carefully for indication of what nursing diagnosis? A. Hypothermia B. Diarrhea C. Ineffective coping D. Imbalanced nutrition: Less than body requirements

Ans: C Rationale: Most clients with hemophilia are diagnosed as children. They often require assistance in coping with the condition because it is chronic, places restrictions on their lives, and is an inherited disorder that can be passed to future generations. Children with hemophilia are not at risk of hypothermia, diarrhea, or imbalanced nutrition.

A medical nurse has admitted four clients over the course of a 12-hour shift. For which client would assessment of ankle-brachial index (ABI) be most clearly warranted? A. A client who has peripheral edema secondary to chronic heart failure B. An older adult client who has a diagnosis of unstable angina C. A client with poorly controlled type 1 diabetes who is a smoker D. A client who has community-acquired pneumonia and a history of COPD

Ans: C Rationale: Nurses should perform a baseline ABI on any client with decreased pulses or any client 50 years of age or older with a history of diabetes or smoking. The other answers do not apply.

The nurse's review of a client's most recent blood work reveals a significant increase in the number of band cells. The nurse's subsequent assessment should focus on which of the following? A. Respiratory function B. Evidence of decreased tissue perfusion C. Signs and symptoms of infection D. Recent changes in activity tolerance

Ans: C Rationale: Ordinarily, band cells account for only a small percentage of circulating granulocytes, although their percentage can increase greatly under conditions in which neutrophil production increases, such as infection. This finding is not suggestive of problems with oxygenation and subsequent activity intolerance.

A nurse is performing an admission assessment on a 40-year-old man who has been admitted for outpatient surgery on the right knee. While taking the client's family history, the client states, "My father died of prostate cancer at age 48." The nurse should instruct the client on which of the following health promotion activities? A. The client will need PSA levels drawn starting at age 55. B. The client should have testing for presence of the CDH1 and STK11 genes. C. The client should have PSA levels drawn regularly. D. The client should limit alcohol use due to the risk of malignancy.

Ans: C Rationale: PSA screening is warranted by the client's family history and should not be delayed until age 55. The CDH1 and STK11 genes do not relate to the risk for prostate cancer. Alcohol consumption by the client should be limited. However, this is not the most important health promotion intervention.

A nurse practitioner is assessing a 55-year-old male client who reports perineal discomfort, burning, urgency, and frequency with urination. The client states that he has pain with ejaculation. The nurse knows that the client is exhibiting symptoms of: A. varicocele. B. epididymitis. C. prostatitis. D. hydrocele.

Ans: C Rationale: Perineal discomfort, burning, urgency, frequency with urination, and pain with ejaculation are indicative of prostatitis. A varicocele is an abnormal dilation of the pampiniform venous plexus and the internal spermatic vein in the scrotum (the network of veins from the testis and the epididymis that constitute part of the spermatic cord). Epididymitis is an infection of the epididymis that usually descends from an infected prostate or urinary tract; it may also develop as a complication of gonorrhea. A hydrocele is a collection of fluid, generally in the tunica vaginalis of the testis, although it may also collect within the spermatic cord.

Diagnostic testing has resulted in a diagnosis of acute myeloid leukemia (AML) in an adult client who is otherwise healthy. The client and the care team have collaborated and the client will soon begin induction therapy. The nurse should prepare the client for: A. daily treatment with targeted therapy medications. B. radiation therapy on a daily basis. C. hematopoietic stem cell transplantation. D. an aggressive course of chemotherapy.

Ans: D Rationale: Attempts are made to achieve remission of AML by the aggressive administration of chemotherapy, called induction therapy, which usually requires hospitalization for several weeks. Induction therapy is not synonymous with radiation, stem cell transplantation, or targeted therapies.

An uncircumcised 78-year-old male client has presented at the clinic reporting that they cannot retract the foreskin over the glans. On examination, it is noted that the foreskin is very constricted. The nurse should recognize the presence of what health problem? A. Bowendisease B. Peyronie disease C. Phimosis D. Priapism

Ans: C Rationale: Phimosis is the term used to describe a condition in which the foreskin is so constricted that it cannot be retracted over the glans. Bowen disease is an in situ carcinoma of the penis. Peyronie disease is an acquired, benign condition that involves the buildup of fibrous plaques in the sheath of the corpus cavernosum. Priapism is an uncontrolled, persistent erection of the penis from either neural or vascular causes, including medications, sickle cell thrombosis, leukemic cell infiltration, spinal cord tumors, and tumor invasion of the penis or its vessels.

How should the nurse best position a client who has leg ulcers that are venous in origin? A. Keep the client's legs flat and straight. B. Keep the client's knees bent to a 45-degree angle and supported with pillows. C. Elevate the client's lower extremities. D. Dangle the client's legs over the side of the bed.

Ans: C Rationale: Positioning of the legs depends on whether the ulcer is of arterial or venous origin. With ulcers of venous origin, the lower extremities should be elevated to avoid dependent edema. Simply bending the knees to a 45-degree angle would not prevent dependent edema, as they must be elevated above the level of the heart. Dangling the client's legs and applying pillows may further compromise venous return.

A client's low hemoglobin level has necessitated transfusion of packed red blood cells. Prior to administration, which action should the nurse perform? A. Have the client identify the blood type in writing. B. Ensure that the client has granted verbal consent for transfusion. C. Assess the client's vital signs to establish baselines. D. Facilitate insertion of a central venous catheter.

Ans: C Rationale: Prior to a transfusion, the nurse must take the client's temperature, pulse, respiration, and blood pressure to establish a baseline. Written consent is required, and the client's blood type is determined by type and cross match, not by the client's self-declaration. Peripheral venous access is sufficient for blood transfusion.

A nurse is caring for a 33-year-old male client who has come to the clinic for a physical examination. The client states not having a routine physical in 5 years. The health care provider's digital rectal examination (DRE) reveals "stony" hardening in the posterior lobe of the prostate gland that is not mobile. The nurse recognizes that the observation typically indicates what? A. A normal finding B. A sign of early prostate cancer C. Evidence of a more advanced lesion D. Metastatic disease

Ans: C Rationale: Routine repeated DRE (preferably by the same examiner) is important, because early cancer may be detected as a nodule within the gland or as an extensive hardening in the posterior lobe. The more advanced lesion is "stony hard" and fixed. This finding is not suggestive of metastatic disease.

A nurse is closely monitoring a client who has recently been diagnosed with an abdominal aortic aneurysm. What assessment finding would signal an impending rupture of the client's aneurysm? A. Sudden increase in blood pressure and a decrease in heart rate B. Cessation of pulsating in an aneurysm that has previously been pulsating visibly C. Sudden onset of severe back or abdominal pain D. New onset of hemoptysis

Ans: C Rationale: Signs of impending rupture include severe back or abdominal pain, which may be persistent or intermittent. Impending rupture is not typically signaled by increased blood pressure, bradycardia, cessation of pulsing, or hemoptysis.

A client has received the news that the client's treatment for Hodgkin lymphoma has been deemed successful and that no further treatment is necessary at this time. The care team should ensure that the client receives regular health assessments in the future due to the risk of which complication? A. Iron-deficiency anemia B. Hemophilia C. Secondary malignancy D. Lymphedema

Ans: C Rationale: Survivors of Hodgkin lymphoma have a high risk of secondary malignancies. There is no consequent risk of anemia, lymphedema, or hemophilia.

The nurse is preparing to administer warfarin to a client with deep vein thrombophlebitis. Which laboratory value would most clearly indicate that the client's warfarin is at therapeutic levels? A. Partial thromboplastin time (PTT) within normal reference range B. Prothrombin time (PT) 8 to 10 times the control C. International normalized ratio (INR) between 2 and 3 D. Hematocrit of 32%

Ans: C Rationale: The INR is most often used to determine whether warfarin is at a therapeutic level; an INR of 2 to 3 is considered therapeutic. Warfarin is also considered to be at therapeutic levels when the client's PT is 1.5 to 2 times the control. Higher values indicate increased risk of bleeding and hemorrhage, whereas lower values indicate increased risk of blood clot formation. Heparin, not warfarin, prolongs PTT. Hematocrit does not provide information on the effectiveness of warfarin; however, a falling hematocrit in a client taking warfarin may be a sign of hemorrhage.

A nursing student is learning how to perform sexual assessments using the PLISSIT model. According to this model, the student should begin an assessment by doing which of the following? A. Briefly teaching the client about normal sexual physiology B. Assuring the client that what they say will be confidential C. Asking the client if the client is willing to discuss sexual functioning D. Ensuring client privacy

Ans: C Rationale: The PLISSIT (permission, limited information, specific suggestions, intensive therapy) model of sexual assessment and intervention may be used to provide a framework for nursing interventions. By beginning with the client's permission, the nurse establishes a client-centered focus.

A client presents to the clinic reporting the inability to grasp objects with the right hand. The client's right arm is cool and has a difference in blood pressure of more than 20 mm Hg compared with the left arm. The nurse should expect that the primary provider may diagnose the client with which health problem? A. Lymphedema B. Raynaud phenomenon C. Upper extremity arterial occlusive disease D. Upper extremity venous thromboembolism (VTE)

Ans: C Rationale: The client with upper extremity arterial occlusive disease typically complains of arm fatigue and pain with exercise (forearm claudication) and inability to hold or grasp objects (e.g., combing hair, placing objects on shelves above the head) and, occasionally, difficulty driving. Assessment findings include coolness and pallor of the affected extremity, decreased capillary refill, and a difference in arm blood pressures of more than 20 mm Hg. These symptoms are not closely associated with Raynaud disease or lymphedema. The upper extremities are rare sites for VTE.

The nurse is admitting a client with a diagnosis of left ventricular hypertrophy. The client reports dyspnea on exertion, as well as fatigue. Which diagnostic tool would be most helpful in diagnosing this type of myopathy? A. Cardiac catheterization B. Arterial blood gases C. Echocardiogram D. Exercise stress test

Ans: C Rationale: The echocardiogram (ECG) is one of the most helpful diagnostic tools because the structure and function of the ventricles can be observed easily. The ECG is also important, and can demonstrate arrhythmias and changes consistent with left ventricular hypertrophy. Cardiac catheterization specifically addresses coronary artery function and arterial blood gases evaluate gas exchange and acid balance. Stress testing is not normally used to differentiate cardiomyopathy from other cardiac pathologies.

A client is undergoing diagnostic testing for mitral stenosis. What statement by the client during the nurse's interview is most suggestive of this valvular disorder? A. "I get chest pain from time to time, but it usually resolves when I rest." B. "Sometimes when I'm resting, I can feel my heart skip a beat." C. "Whenever I do any form of exercise I get terribly short of breath." D. "My feet and ankles have gotten terribly puffy the last few weeks."

Ans: C Rationale: The first symptom of mitral stenosis is often breathing difficulty (dyspnea) on exertion as a result of pulmonary venous hypertension. Clients with mitral stenosis are likely to show progressive fatigue as a result of low cardiac output. Palpitations occur in some clients, but dyspnea is a characteristic early symptom. Peripheral edema and chest pain are atypical.

Which factor is the most common cause of mitral valve regurgitation in developing countries? A. A decrease in gamma globulins B. An insect bite C. Rheumatic heart disease and its sequelae D. Sepsis and its sequelae

Ans: C Rationale: The most common cause of mitral valve regurgitation in developing countries is rheumatic heart disease and its sequelae, not a decrease in gamma globulins, an insect bite, or sepsis and its sequelae.

A nurse is providing education to a client with iron deficiency anemia who has been prescribed iron supplements. What should the nurse include in health education? A. Take the iron with dairy products to enhance absorption. B. Increase the intake of vitamin E to enhance absorption. C. Iron will cause the stools to darken in color. D. Limit foods high in fiber due to the risk for diarrhea.

Ans: C Rationale: The nurse will inform the client that iron will cause the stools to become dark in color. Iron should be taken on an empty stomach, as its absorption is affected by food, especially dairy products. Clients should be instructed to increase their intake of vitamin C to enhance iron absorption. Foods high in fiber should be consumed to minimize problems with constipation, a common side effect associated with iron therapy.

The nurse is creating a plan of care for a client with cardiomyopathy. Which goal would be a priority for the client? A. Absence of complications B. Adherence to the self-care program C. Improved cardiac output D. Increased activity tolerance

Ans: C Rationale: The priority nursing diagnosis of a client with cardiomyopathy would include improved or maintained cardiac output. Regardless of the category and cause, cardiomyopathy may lead to severe heart failure, lethal dysrhythmias, and death. The pathophysiology of all cardiomyopathies is a series of progressive events that culminate in impaired cardiac output. Absence of complications, adherence to the self-care program, and increased activity tolerance should be included in the care plan, but they do not have the priority of improved cardiac output.

The clinical nurse educator is presenting health promotion education to a client who will be treated for non-Hodgkin lymphoma on an outpatient basis. The nurse should recommend which of the following actions? A. Avoiding direct sun exposure in excess of 15 minutes daily B. Avoiding grapefruit juice and fresh grapefruit C. Avoiding highly crowded public places D. Using an electric shaver rather than a razor

Ans: C Rationale: The risk of infection is significant for these clients, not only from treatment-related myelosuppression but also from the defective immune response that results from the disease itself. Limiting infection exposure is thus necessary. The need to avoid grapefruit is dependent on the client's medication regimen. Sun exposure and the use of razors are not necessarily contraindicated.

A nurse is planning the care of a client with a diagnosis of sickle cell disease who has been admitted for the treatment of an acute vaso-occlusive crisis. Which nursing diagnosis should the nurse prioritize in the client's plan of care? A. Risk for disuse syndrome related to ineffective peripheral circulation B. Functional urinary incontinence related to urethral occlusion C. Ineffective tissue perfusion related to thrombosis D. Ineffective thermoregulation related to hypothalamic dysfunction

Ans: C Rationale: There are multiple potential complications of sickle cell disease and sickle cell crises. Central among these, however, is the risk of thrombosis and consequent lack of tissue perfusion. Sickle cell crises are not normally accompanied by impaired thermoregulation or genitourinary complications. Risk for disuse syndrome is not associated with the effects of acute vaso-occlusive crisis.

The nurse is caring for a client with acute pericarditis. Which nursing management would be instituted to minimize complications? A. The nurse keeps the client isolated to prevent nosocomial infections. B. The nurse encourages coughing and deep breathing. C. The nurse helps the client with activities until the pain and fever subside. D. The nurse encourages increased fluid intake until the infection resolves.

Ans: C Rationale: To minimize complications, the nurse helps the client with activity restrictions until the pain and fever subside. As the client's condition improves, the nurse encourages gradual increases of activity. Actions to minimize complications of acute pericarditis do not include keeping the client isolated. Due to pain, coughing and deep breathing are not normally encouraged. An increase in fluid intake is not always necessary.

A nurse in a long-term care facility is admitting a new resident who has a bleeding disorder. When planning this resident's care, the nurse should include which action? A. Housing the resident in a private room B. Implementing a passive ROM program C. Implementing of a plan for fall prevention D. Providing the client with a high-fiber diet

Ans: C Rationale: To prevent bleeding episodes, the nurse should ensure that an older adult with a bleeding disorder does not suffer a fall. Activity limitation is not necessarily required, however. A private room is not necessary and there is no reason to increase fiber intake.

A nurse is admitting a client to the medical unit who has a history of peripheral artery disease (PAD). While providing the health history, the client reports smoking about two packs of cigarettes a day, having a history of alcohol abuse, and not exercising. Which topic would be the priority health education for this client? A. The lack of exercise, which is the main cause of PAD B. The likelihood that heavy alcohol intake is a significant risk factor for PAD C. The nicotine in cigarettes, which is a powerful vasoconstrictor and may cause or aggravate PAD D. Alcohol, which suppresses the immune system, creates high glucose levels, and may cause PAD

Ans: C Rationale: Tobacco is a powerful vasoconstrictor; its use with PAD is highly detrimental, and clients are strongly advised to stop using tobacco. Sedentary lifestyle is also a risk factor, but smoking is likely a more significant risk factor that the nurse should address. Alcohol use is less likely to cause PAD, although it carries numerous health risks.

A client with a history of atrial fibrillation has contacted the clinic reporting an accidental overdose on prescribed warfarin. The nurse should recognize the possible need for which antidote? A. Intravenous immunoglobulins (IVIG) B. Factor IX C. Vitamin K D. Factor VIII

Ans: C Rationale: Vitamin K is given as an antidote for warfarin toxicity. IVIG is a form of immunosuppressive therapy given to treat immune thrombocytopenic purpura and to counteract hemolytic transfusion reaction and neutralizing antibodies (inhibitors) that develop in response to factor replacement therapy in clients with hemophilia. IVIG is not used as an antidote for warfarin toxicity. Factors VIII and IX are clotting factors that are deficient in clients with hemophilia due to a genetic defect; these clients may receive recombinant forms of these factors to treat their condition.

A 17-year-old client is being treated in the intensive care unit after going into cardiac arrest during a football practice. Diagnostic testing reveals cardiomyopathy as the cause of the arrest. Which type of cardiomyopathy is particularly common among young people who appear otherwise healthy? A. Dilated cardiomyopathy (DCM) B. Arrhythmogenic right ventricular cardiomyopathy (ARVC) C. Hypertrophic cardiomyopathy (HCM) D. Restrictive or constrictive cardiomyopathy (RCM)

Ans: C Rationale: With HCM, cardiac arrest (i.e., sudden cardiac death) may be the initial manifestation in young people, including athletes. DCM, ARVC, and RCM are not typically present in younger adults who appear otherwise healthy.

A nurse is admitting a client with immune thrombocytopenic purpura to the unit. In completing the admission assessment, the nurse must be alert for what medications that potentially alter platelet function? Select all that apply. A. Antihypertensives B. Penicillins C. Sulfa-containing medications D. Aspirin-based drugs E. NSAIDs

Ans: C, D, E Rationale: The nurse must be alert for sulfa-containing medications and others that alter platelet function (e.g., aspirin-based or other NSAIDs). Antihypertensive drugs and the penicillins do not alter platelet function.

A nurse is working with a client who has a diagnosis of Cushing syndrome. When completing a physical assessment, the nurse should specifically observe for what integumentary manifestation? A. Alopecia B. Yellowish skin tone C. Patchy, bronze pigmentation D. Hirsutism

Ans: D Rationale: Cushing syndrome causes excessive hair growth, especially in women. Alopecia is hair loss from the scalp and other parts of the body. Jaundice causes a yellow discoloration in light-skinned clients, but this does not accompany Cushing syndrome. Clients that have Addison disease exhibit a bronze discoloration to their skin due to increased melanin production.

A dark-skinned client is admitted to the medical unit with liver disease. To correctly assess this client for jaundice, on what body area should the nurse look for yellow discoloration? A. Elbows B. Lips C. Nail beds D. Sclerae

Ans: D Rationale: Jaundice, a yellowing of the skin, is directly related to elevations in serum bilirubin and is often first observed in the sclerae and mucous membranes.

A nurse is providing an educational presentation addressing the topic of "Protecting Your Skin." When discussing the anatomy of the skin, the nurse should state that what cells are responsible for producing the pigmentation of the skin? A. Islets of Langerhans B. Squamous cells C. T cells D. Melanocytes

Ans: D Rationale: Melanocytes are the special cells of the epidermis that are primarily responsible for producing the pigment melanin. Islets of Langerhans are clusters of cells in the pancreas. Squamous cells are flat, scaly epithelial cells. T cells function in the immune response.

A client with an exceptionally low body mass index has been admitted to the emergency department with signs and symptoms of hypothermia. The nurse should know that this client's susceptibility to heat loss is related to atrophy of what skin component? A. Epidermis B. Merkel cells C. Dermis D. Subcutaneous tissue

Ans: D Rationale: The subcutaneous tissues and the amount of fat deposits are important factors in body temperature regulation. The epidermis is an outermost layer of stratified epithelial cells. Merkel cells are receptors that transmit stimuli to the axon through a chemical synapse. The dermis makes up the largest portion of the skin, providing strength and structure. It is composed of two layers: papillary and reticular.

While assessing a dark-skinned client at the clinic, the nurse notes the presence of patchy, milky-white spots. The nurse knows that this finding is characteristic of what diagnosis? A. Cyanosis B. Addison disease C. Polycythemia D. Vitiligo

Ans: D Rationale: With cyanosis, nail beds are dusky. With polycythemia, the nurse notes ruddy blue face, oral mucosa, and conjunctiva. A bronzed appearance, or "external tan," is associated with Addison disease. Vitiligo is a condition characterized by destruction of the melanocytes in circumscribed areas of skin and appears in light or dark skin as patchy, milky-white spots, often symmetric bilaterally.

An 82-year-old client is being treated in the hospital for a sacral pressure ulcer. What age-related change is most likely to affect the client's course of treatment? A. Increased thickness of the subcutaneous skin layer B. Increased vascular supply to superficial skin layers C. Changes in the character and quantity of bacterial skin flora D. Increased time required for wound healing

Ans: D Rationale: Wound healing becomes slower with age, requiring more time for older adults to recover from surgical and traumatic wounds. There are no changes in skin flora with increased age. Vascular supply and skin thickness both decrease with age.

A client with von Willebrand disease (vWD) has experienced recent changes in bowel function that suggest the need for a screening colonoscopy. What intervention should be performed in anticipation of this procedure? A. The client should not undergo the normal bowel cleansing protocol prior to the procedure. B. The client should receive a unit of fresh-frozen plasma 48 hours before the procedure. C. The client should be admitted to the surgical unit on the day before the procedure. D. The client should be given necessary clotting factors before the procedure.

Ans: D Rationale: A goal of treating vWD is to replace the deficient protein (e.g., vWF or factor VIII) prior to an invasive procedure to prevent subsequent bleeding. Bowel cleansing is not contraindicated and FFP does not reduce the client's risk of bleeding. There may or may not be a need for preprocedure hospital admission.

A client has been living with a diagnosis of anemia for several years and has experienced recent declines in hemoglobin levels despite active treatment. Which assessment finding would signal complications of anemia? A. Venous ulcers and visual disturbances B. Fever and signs of hyperkalemia C. Epistaxis and gastroesophageal reflux D. Shortness of breath and peripheral edema

Ans: D Rationale: A significant complication of anemia is heart failure from chronic diminished blood volume and the heart's compensatory effort to increase cardiac output. Clients with anemia should be assessed for signs and symptoms of heart failure, including dyspnea and peripheral edema. None of the other listed signs and symptoms is characteristic of heart failure.

A nurse is caring for a client whose diagnosis of multiple myeloma is being treated with bortezomib. The nurse should assess for what adverse effect of this treatment? A. Stomatitis B. Nephropathy C. Cognitive changes D. Peripheral neuropathy

Ans: D Rationale: A significant toxicity associated with the use of bortezomib for multiple myeloma is peripheral neuropathy. Stomatitis, cognitive changes, and nephropathy are not noted to be adverse effects of this medication.

A critical care nurse is caring for a client with immune hemolytic anemia. The client is not responding to conservative treatments, and the client's condition is now becoming life-threatening. The nurse is aware that a treatment option in this case may include which intervention? A. Hepatectomy. B. Vitamin K administration. C. Platelet transfusion. D. Splenectomy.

Ans: D Rationale: A splenectomy may be the course of treatment if autoimmune hemolytic anemia does not respond to conservative treatment. Vitamin K administration is treatment for vitamin K deficiency and does not resolve anemia. Platelet transfusion may be the course of treatment for some bleeding disorders. Hepatectomy would not help the client.

A client has returned to the floor after undergoing a transurethral resection of the prostate (TURP). The client has a continuous bladder irrigation system in place. The client reports bladder spasms. What is the most appropriate nursing action to relieve the discomfort of the client? A. Apply a cold compress to the pubic area. B. Notify the urologist promptly. C. Irrigate the catheter with 30 to 50 mL of normal saline as ordered. D. Administer a smooth muscle relaxant as ordered.

Ans: D Rationale: Administering a medication that relaxes smooth muscles can help relieve bladder spasms. Neither a cold compress nor catheter irrigation will alleviate bladder spasms. In most cases, this problem can be relieved without the involvement of the urologist, who will normally order medications on a PRN basis.

The critical care nurse is caring for a client who is receiving cyclosporine postoperative heart transplant. What outcome represents a therapeutic outcome of this pharmacologic treatment? A. The client remains free of thrombus formation. B. The client maintains adequate cardiac output. C. The client has an increase in white cell count. D. The client does not experience organ rejection.

Ans: D Rationale: After heart transplant, clients are constantly balancing the risk of rejection with the risk of infection. Most commonly, clients receive medications such as cyclosporine to minimize rejection. Cyclosporine does not prevent thrombus formation, enhance cardiac output, or increase white cell counts.

A client is a candidate for percutaneous balloon valvuloplasty, but is concerned about how this procedure will affect the client's busy work schedule. Which guidance would the nurse provide to the client? A. "Clients generally stay in the hospital for 6 to 8 days." B. "Clients are kept in the hospital until they are independent with all aspects of their care." C. "Clients need to stay in the hospital until they regain normal heart function for their age." D. "Clients usually remain at the hospital for 24 to 48 hours."

Ans: D Rationale: After undergoing percutaneous balloon valvuloplasty, the client usually remains in the hospital for 24 to 48 hours. Prediagnosis levels of heart function are not always attainable and the client does not need to be wholly independent prior to discharge.

A client with a history of rheumatic heart disease knows that the client is at risk for bacterial endocarditis when undergoing invasive procedures. Prior to a scheduled cystoscopy, the nurse should ensure that the client knows the importance of taking which drug? A. Enoxaparin B. Metoprolol C. Azathioprine D. Amoxicillin

Ans: D Rationale: Although rare, bacterial endocarditis may be life threatening. A key strategy is primary prevention in high-risk clients (i.e., those with rheumatic heart disease, mitral valve prolapse, or prosthetic heart valves). Antibiotic prophylaxis is recommended for high-risk clients immediately before and sometimes after certain procedures. Amoxicillin is the drug of choice. None of the other listed drugs is an antibiotic.

Graduated compression stockings have been prescribed to treat a client's venous insufficiency. What education should the nurse prioritize when introducing this intervention to the client? A. The need to take anticoagulants concurrent with using compression stockings B. The need to wear the stockings on a "one day on, one day off" schedule C. The importance of wearing the stockings around the clock to ensure maximum benefit D. The importance of ensuring the stockings are applied evenly with no pressure points

Ans: D Rationale: Any type of stocking can inadvertently become a tourniquet if applied incorrectly (i.e., rolled tightly at the top). In such instances, the stockings produce rather than prevent stasis. For ambulatory clients, graduated compression stockings are removed at night and reapplied before the legs are lowered from the bed to the floor in the morning. They are used daily, not on alternating days. Anticoagulants are not always indicated in clients who are using compression stockings.

The nurse is caring for a client with right ventricular hypertrophy and, consequently, decreased right ventricular function. What valvular disorder may have contributed to this client's diagnosis? A. Mitral valve regurgitation B. Aortic stenosis C. Aortic regurgitation D. Mitral valve stenosis

Ans: D Rationale: Because no valve protects the pulmonary veins from the backward flow of blood from the atrium, the pulmonary circulation becomes congested. As a result, the right ventricle must contract against an abnormally high pulmonary arterial pressure and is subjected to excessive strain. Eventually, the right ventricle fails. None of the other listed valvular disorders has this pathophysiological effect.

A client is being treated for polycythemia vera, and the nurse is providing health education. Which practice should the nurse recommend to prevent the complications of this health problem? A. Avoiding natural sources of vitamin K B. Avoiding altitudes of 1500 feet (457 meters) C. Performing active range of motion exercises daily D. Avoiding tight and restrictive clothing on the legs

Ans: D Rationale: Because of the risk of deep vein thrombosis, clients with polycythemia vera should avoid tight and restrictive clothing. There is no need to avoid foods with vitamin K or to avoid higher altitudes. Activity levels should be maintained, but there is no specific need for range of motion exercises.

A client has just been diagnosed with prostate cancer and is scheduled for brachytherapy the following week. The client and spouse are unsure of having the procedure because their child is 3 months' pregnant. What is the most appropriate teaching the nurse should provide to this family? A. The client should not be in contact with the baby after delivery. B. The client's treatment poses no risk to the child or the infant. C. The client's brachytherapy may be contraindicated for safety reasons. D. The client should avoid close contact with the child for 2 months.

Ans: D Rationale: Brachytherapy involves the implantation of interstitial radioactive seeds under anesthesia. The surgeon uses ultrasound guidance to place about 80 to 100 seeds, and the client returns home after the procedure. Exposure of others to radiation is minimal, but the client should avoid close contact with pregnant women and infants for up to 2 months.

A nurse is planning discharge health education for a client who will soon undergo placement of a mechanical valve prosthesis. Which topic would the nurse prioritize? A. The need for long-term antibiotics B. The need for 7 to 10 days of bed rest C. Strategies for preventing atherosclerosis D. Strategies for infection prevention

Ans: D Rationale: Clients with a mechanical valve prosthesis (including annuloplasty rings and other prosthetic materials used in valvuloplasty) require education to prevent infective endocarditis. Despite these infection risks, antibiotics are not used long term. Activity management is important, but extended bed rest is unnecessary. Valve replacement does not create a heightened risk for atherosclerosis.

A client lives with a diagnosis of sickle cell disease and receives frequent blood transfusions. The nurse should recognize the client's consequent risk of what complication of treatment? A. Hypovolemia B. Vitamin B12 deficiency C. Thrombocytopenia D. Iron overload

Ans: D Rationale: Clients with chronic transfusion requirements can quickly acquire more iron than they can use, leading to iron overload. These individuals are not at risk for hypovolemia and there is no consequent risk for low platelet or vitamin B12 levels.

A client has been diagnosed with erectile dysfunction; the cause has been determined to be psychogenic. The client's interdisciplinary plan of care should prioritize which of the following interventions? A. Penile implant B. PDE-5 inhibitors C. Physical therapy D. Psychotherapy

Ans: D Rationale: Clients with erectile dysfunction from psychogenic causes are referred to a health care provider or therapist who specializes in sexual dysfunction. Because of the absence of an organic cause, medications and penile implants are not first-line treatments. Physical therapy is not normally effective in the treatment of ED.

A client is being treated on the medical unit for a sickle cell crisis. The nurse's most recent assessment reveals a fever and a new onset of fine crackles on lung auscultation. Which action by the nurse would be the most appropriate? A. Apply supplementary oxygen by nasal cannula. B. Administer bronchodilators by nebulizer. C. Liaise with the respiratory therapist and consider high-flow oxygen. D. Inform the healthcare provider that the client may have an infection.

Ans: D Rationale: Clients with sickle cell disease are highly susceptible to infection, thus any early signs of infection should be reported promptly. There is no evidence of respiratory distress, so oxygen therapy and bronchodilators are not indicated.

A nurse is planning the postoperative care of a client who is scheduled for radical prostatectomy. What intraoperative position will place the client at particular risk for the development of deep vein thrombosis postoperatively? A. Fowler position B. Prone position C. Supine position D. Lithotomy position

Ans: D Rationale: Elastic compression stockings are applied before surgery and are particularly important for prevention of deep vein thrombosis if the client is placed in a lithotomy position during surgery. During a prostatectomy, the client is not placed in the supine, prone, or Fowler position.

An adult client has been diagnosed with iron-deficiency anemia. What nursing diagnosis is most likely to apply to this client's health status? A. Risk for deficient fluid volume related to impaired erythropoiesis B. Risk for infection related to tissue hypoxia C. Acute pain related to uncontrolled hemolysis D. Fatigue related to decreased oxygen-carrying capacity

Ans: D Rationale: Fatigue is the major assessment finding common to all forms of anemia. Anemia does not normally result in acute pain or fluid deficit. The client may have an increased risk of infection due to impaired immune function, but fatigue is more likely.

A home health nurse is caring for a client with multiple myeloma. What intervention should the nurse prioritize when addressing the client's severe bone pain? A. Implementing distraction techniques B. Educating the client about the effective use of hot and cold packs C. Teaching the client to use NSAIDs effectively D. Helping the client manage the opioid analgesic regimen

Ans: D Rationale: For severe pain resulting from multiple myeloma, opioids are likely necessary. NSAIDs would likely be ineffective and are associated with significant adverse effects. Hot and cold packs as well as distraction would be insufficient for severe pain, though they may be useful as adjuncts.

A client has presented at the clinic with symptoms of benign prostatic hyperplasia. What diagnostic findings would suggest that this client has chronic urinary retention? A. Hypertension B. Peripheral edema C. Tachycardia and other dysrhythmias D. Increased blood urea nitrogen (BUN)

Ans: D Rationale: Hypertension, edema, and tachycardia would not normally be associated with benign prostatic hyperplasia. Azotemia is an accumulation of nitrogenous waste products, and kidney injury can occur with chronic urinary retention and large residual volumes.

The nurse educating a client with anemia is describing the process of red blood cell production. When the client's kidneys sense a low level of oxygen in circulating blood, which physiologic response is initiated? A. Increased stem cell synthesis B. Decreased respiratory rate C. Arterial vasoconstriction D. Increased levels of erythropoietin

Ans: D Rationale: If the kidney detects low levels of oxygen, as occurs when fewer red blood cells are available to bind oxygen (i.e., anemia), erythropoietin levels increase. The body does not compensate with vasoconstriction, decreased respiration, or increased stem cell activity.

A 35-year-old client is admitted to the hospital reporting severe headaches, vomiting, and testicular pain. The client's blood work shows reduced numbers of platelets, leukocytes, and erythrocytes, with a high proportion of immature cells. The nurse caring for this client suspects which diagnosis? A. Acute myeloid leukemia (AML) B. Chronic myeloid leukemia (CML) C. Myelodysplastic syndromes (MDS) D. Acute lymphocytic leukemia (ALL)

Ans: D Rationale: In acute lymphocytic leukemia (ALL), manifestations of leukemic cell infiltration into other organs are more common than with other forms of leukemia, and include pain from an enlarged liver or spleen, as well as bone pain. The central nervous system is frequently a site for leukemic cells; thus, clients may exhibit headache and vomiting because of meningeal involvement. Other extranodal sites include the testes and breasts. All the listed types of leukemia, depending on severity and stage, can have the same blood work results. The difference is the client's signs and symptoms, which are closely associated with ALL. A large number of clients when first diagnosed with any type of leukemia are asymptomatic or have nonspecific symptoms. It is discovered on routine lab work.

The nurse is caring for a client who has developed scar tissue in many of the areas that normally produce blood cells. What organs can become active in blood cell production by the process of extramedullary hematopoiesis? A. Spleen and kidneys B. Kidneys and pancreas C. Pancreas and liver D. Liver and spleen

Ans: D Rationale: In adults with disease that causes marrow destruction, fibrosis, or scarring, the liver and spleen can also resume production of blood cells by a process known as extramedullary hematopoiesis. The kidneys and pancreas do not produce blood cells for the body.

A client comes into the clinic reporting fatigue. Blood work shows an increased bilirubin concentration and an increased reticulocyte count. Which condition should the nurse most suspect the client has? A. A hypoproliferative anemia. B. A leukemia. C. Thrombocytopenia. D. A hemolytic anemia.

Ans: D Rationale: In hemolytic anemias, premature destruction of erythrocytes results in the liberation of hemoglobin from the erythrocytes into the plasma; the released hemoglobin is converted in large part to bilirubin, and therefore the bilirubin concentration rises. The increased erythrocyte destruction leads to tissue hypoxia, which in turn stimulates erythropoietin production. This increased production is reflected in an increased reticulocyte count as the bone marrow responds to the loss of erythrocytes. Hypoproliferative anemias, leukemia, and thrombocytopenia lack this pathology and presentation.

A nurse working in a long-term care facility is performing the admission assessment of a newly admitted 85-year-old resident. During inspection of the resident's feet, the nurse notes early evidence of gangrene on one of the resident's great toes. The nurse should assess for further evidence of which health problem? A. Chronic venous insufficiency B. Raynaud phenomenon C. Venous thromboembolism (VTE) D. Peripheral artery disease (PAD)

Ans: D Rationale: In older adults, symptoms of PAD may be more pronounced than in younger people. In older adult clients who are inactive, gangrene may be the first sign of disease. Venous insufficiency does not normally manifest with gangrene. Similarly, VTE and Raynaud phenomenon do not cause the ischemia that underlies gangrene.

When teaching a client with sickle cell disease about strategies to prevent crises, what measures should the nurse recommend? A. Using prophylactic antibiotics and performing meticulous hygiene B. Maximizing physical activity and taking OTC iron supplements C. Limiting psychosocial stress and eating a high-protein diet D. Avoiding cold temperatures and ensuring sufficient hydration

Ans: D Rationale: Keeping warm and providing adequate hydration can be effective in diminishing the occurrence and severity of attacks. Hygiene, antibiotics, and high protein intake do not prevent crises. Maximizing activity may exacerbate pain and be unrealistic.

A client comes to the walk-in clinic with reports of pain in the foot following stepping on a roofing nail 4 days ago. The client has a visible red streak running up his foot and ankle. Which health problem should the nurse suspect? A. Cellulitis B. Local inflammation C. Elephantiasis D. Lymphangitis

Ans: D Rationale: Lymphangitis is an acute inflammation of the lymphatic channels. It arises most commonly from a focus of infection in an extremity. Usually, the infectious organism is hemolytic streptococcus. The characteristic red streaks that extend up the arm or the leg from an infected wound outline the course of the lymphatic vessels as they drain. Cellulitis is caused by bacteria, which cause a generalized edema in the subcutaneous tissues surrounding the affected area. Local inflammation would not present with red streaks in the lymphatic channels. Elephantiasis is transmitted by mosquitoes that carry parasitic worm larvae; the parasites obstruct the lymphatic channels and results in gross enlargement of the limbs.

A nurse is planning the care of a client who has a diagnosis of hemophilia A. When addressing the nursing diagnosis of Acute Pain Related to Joint Hemorrhage, what principle should guide the nurse's choice of interventions? A. Gabapentin (Neurontin) is effective because of the neuropathic nature of the client's pain. B. Opioids partially inhibit the client's synthesis of clotting factors. C. Opioids may cause vasodilation and exacerbate bleeding. D. NSAIDs are contraindicated due to the risk for bleeding.

Ans: D Rationale: NSAIDs may be contraindicated in clients with hemophilia due to the associated risk of bleeding. Opioids do not have a similar effect and they do not inhibit platelet synthesis. The pain associated with hemophilia is not neuropathic.

A client confides to the nurse that he cannot engage in sexual activity. The client is 27 years old and has no apparent history of chronic illness that would contribute to erectile dysfunction. What does the nurse know will be ordered for this client to assess sexual functioning? A. Sperm count B. Ejaculation capacity tests C. Engorgement tests D. Nocturnal penile tumescence tests

Ans: D Rationale: Nocturnal penile tumescence tests may be conducted in a sleep laboratory to monitor changes in penile circumference during sleep using various methods to determine number, duration, rigidity, and circumference of penile erections; the results help identify whether the erectile dysfunction is caused by physiologic and/or psychological factors. A sperm count would be done if the client was experiencing infertility. Ejaculation capacity tests and engorgement tests are not applicable for assessment in this circumstance.

The nurse is assessing a woman who is pregnant at 27 weeks' gestation. The client is concerned about the recent emergence of varicose veins on the backs of her calves. What is the nurse's best action? A. Facilitate a referral to a vascular surgeon. B. Assess the client's ankle-brachial index (ABI) and perform Doppler ultrasound testing. C. Encourage the client to increase her activity level. D. Teach the client that circulatory changes during pregnancy frequently cause varicose veins.

Ans: D Rationale: Pregnancy may cause varicosities because of hormonal effects related to decreased venous outflow, increased pressure by the gravid uterus, and increased blood volume. In most cases, no intervention or referral is necessary. This finding is not an indication for ABI assessment and increased activity will not likely resolve the problem.

A 25-year-old client comes to the emergency department with excessive bleeding from a cut sustained when cleaning a knife. Blood work shows a prolonged prothrombin time (PT), but a vitamin K deficiency is ruled out. When assessing the client, areas of ecchymosis are noted on other areas of the body. Which of the following is the most plausible cause of the client's signs and symptoms? A. Lymphoma B. Leukemia C. Hemophilia D. Hepatic dysfunction

Ans: D Rationale: Prolongation of the PT, unless it is caused by vitamin K deficiency, may indicate severe hepatic dysfunction. Liver dysfunction can lead to decreased amount of factors needed for coagulation and hemostasis. The majority of hemophiliacs are diagnosed as children. The scenario does not describe signs or symptoms of lymphoma or leukemia.

An intensive care nurse is aware of the need to identify clients who may be at risk of developing disseminated intravascular coagulation (DIC). Which ICU client most likely faces the highest risk of DIC? A. A client with extensive burns B. A client who has a diagnosis of acute respiratory distress syndrome C. A client who suffered multiple trauma in a workplace accident D. A client who is being treated for septic shock

Ans: D Rationale: Sepsis is a common cause of DIC. A wide variety of acute illnesses can precipitate DIC, but sepsis is specifically identified as a cause.

A 57-year-old male client reports that when having an erection the penis curves and becomes painful. The client's diagnosis is identified as severe Peyronie disease. The nurse should prepare the client for what likely treatment modality? A. Physical therapy B. Treatment with PDE-5 inhibitors C. Intracapsular hydrocortisone injections D. Surgery

Ans: D Rationale: Surgical removal of mature plaques is used to treat severe Peyronie disease. There is no potential benefit to physical therapy and hydrocortisone injections are not normally used. PDE-5 inhibitors would exacerbate the problem.

A public health nurse has been asked to provide a health promotion session for men at a wellness center. What should the nurse inform the participants about testicular cancer? A. It is most common among men over 55. B. It is one of the least curable solid tumors. C. It typically does not metastasize. D. It is highly responsive to treatment.

Ans: D Rationale: Testicular cancer is most common among men 15 to 35 years of age and produces a painless enlargement of the testicle. Testicular cancers metastasize early but are one of the most curable solid tumors, being highly responsive to chemotherapy.

A client's diagnosis of atrial fibrillation has prompted the primary care provider to prescribe warfarin. When assessing the therapeutic response to this medication, which action by the nurse is the most appropriate? A. Assess for signs of myelosuppression. B. Review the client's platelet level. C. Assess the client's capillary refill time. D. Review the client's international normalized ratio (INR).

Ans: D Rationale: The INR and activated partial thromboplastin time serve as useful tools for evaluating a client's clotting ability and monitoring the therapeutic effectiveness of anticoagulant medications. The client's platelet level is not normally used as a short-term indicator of anticoagulation effectiveness. Assessing the client for signs of myelosuppression and assessing capillary refill time do not address the effectiveness of anticoagulants.

A nurse is reviewing the physiologic factors that affect a client's cardiovascular health and tissue oxygenation. What is the systemic arteriovenous oxygen difference? A. The average amount of oxygen removed by each organ in the body B. The amount of oxygen removed from the blood by the heart C. The amount of oxygen returning to the lungs via the pulmonary artery D. The amount of oxygen in aortic blood minus the amount of oxygen in the venacaval blood

Ans: D Rationale: The average amount of oxygen removed collectively by all of the body tissues is about 25%. This means that the blood in the vena cava contains about 25% less oxygen than aortic blood. This is known as the systemic arteriovenous oxygen difference. The other answers do not apply.

Two units of packed red blood cells have been prescribed for a client who has experienced a gastrointestinal bleed. The client is highly reluctant to receive a transfusion, stating, "I'm terrified of getting acquired immunodeficiency syndrome (AIDS) from a blood transfusion." How can the nurse best address the client's concerns? A. "All donated blood is treated with antiretroviral medications before it is used." B. "That did happen in some high-profile cases in the 20th century, but it is no longer a possibility." C. "HIV was eradicated from the blood supply in the early 2000s." D. "Donated blood is screened for human immunodeficiency virus (HIV), and the risk of contraction is very low."

Ans: D Rationale: The client can be reassured about the very low possibility of contracting HIV from the transfusion. However, it is not an absolute impossibility. Antiretroviral medications are not introduced into donated blood. The blood supply is constantly dynamic, due to the brief life of donated blood.

The nurse caring for a client with a leg ulcer has finished assessing the client and is developing a problem list prior to writing a plan of care. What priority risk would the care plan address? A. Disuse syndrome B. Ineffective health maintenance C. Sedentary lifestyle D. Insufficient nutrition

Ans: D Rationale: The client with leg ulcers is at risk for insufficient nutrition related to the increased need for nutrients that promote wound healing. The risk for disuse syndrome is a state in which an individual is at risk for deterioration of body systems owing to prescribed or unavoidable musculoskeletal inactivity. A leg ulcer will affect activity, but rarely to this degree. Leg ulcers are not necessarily a consequence of ineffective health maintenance or a sedentary lifestyle.

A clinic nurse is working with a client who has a long-standing diagnosis of polycythemia vera. How can the nurse best gauge the course of the client's disease? A. Document the color of the client's palms and face during each visit. B. Follow the client's erythrocyte sedimentation rate over time. C. Document the client's response to erythropoietin injections. D. Follow the trends of the client's hematocrit.

Ans: D Rationale: The course of polycythemia vera can be best ascertained by monitoring the client's hematocrit, which should remain below 45%. Erythropoietin injections would exacerbate the condition. Skin tone should be observed, but is a subjective assessment finding. The client's erythrocyte sedimentation rate is not relevant to the course of the disease.

An interdisciplinary team has been commissioned to create policies and procedures aimed at preventing acute hemolytic transfusion reactions. What action has the greatest potential to reduce the risk of this transfusion reaction? A. Ensure that blood components are never infused at a rate greater than 125 mL/h. B. Administer prophylactic antihistamines prior to all blood transfusions. C. Establish baseline vital signs for all clients receiving transfusions. D. Be vigilant in identifying the client and the blood component.

Ans: D Rationale: The most common causes of acute hemolytic reaction are errors in blood component labeling and client identification that result in the administration of an ABO-incompatible transfusion. Actions to address these causes are necessary in all health care settings. Prophylactic antihistamines are not normally given, and would not prevent acute hemolytic reactions. Similarly, baseline vital signs and slow administration will not prevent this reaction.

A nurse is caring for a client who is undergoing preliminary testing for a hematologic disorder. Which sign or symptom of a hematologic disorder is most common? A. Sudden change in level of consciousness (LOC) B. Recurrent infections C. Anaphylaxis D. Severe fatigue

Ans: D Rationale: The most common indicator of hematologic disease is extreme fatigue. This is more common than changes in LOC, infections, or anaphylaxis.

A client who has recently recovered from a systemic viral infection is undergoing diagnostic testing for myocarditis. Which of the nurse's assessment findings is most consistent with myocarditis? A. Sudden changes in level of consciousness (LOC) B. Peripheral edema and pulmonary edema C. Pleuritic chest pain D. Flu-like symptoms

Ans: D Rationale: The most common symptoms of myocarditis are flu-like. Chest pain, edema, and changes in LOC are not characteristic of myocarditis.

A client with Hodgkin lymphoma is receiving information from the oncology nurse. The client asks the nurse why it is necessary to stop drinking and smoking and stay out of the sun. Which response by the nurse would be best? A. "Avoiding these factors can reduce the risk of Reed-Sternberg cells developing." B. "These behaviors can reduce the effectiveness of your chemotherapy." C. "Engaging in these activities increases your risk of hemorrhage." D. "It's important to reduce other factors that increase the risk of second cancers."

Ans: D Rationale: The nurse should encourage clients to reduce other factors that increase the risk of developing second cancers, such as use of tobacco and alcohol and exposure to environmental carcinogens and excessive sunlight. The presence of Reed-Sternberg cells is the pathologic hallmark and essential diagnostic criterion for Hodgkin lymphoma, so avoiding these behaviors will not reduce the risk of Reed-Sternberg cells developing. There is no evidence that these behaviors will reduce the effectiveness of chemotherapy or increase the risk of hemorrhage, which is not a typical complication of Hodgkin lymphoma.

A client's wound has begun to heal and the blood clot which formed is no longer necessary. When a blood clot is no longer needed, the fibrinogen and fibrin will be digested by which of the following? A. Plasminogen B. Thrombin C. Prothrombin D. Plasmin

Ans: D Rationale: The substance plasminogen is required to lyse (break down) the fibrin. Plasminogen, which is present in all body fluids, circulates with fibrinogen and is therefore incorporated into the fibrin clot as it forms. When the clot is no longer needed (e.g., after an injured blood vessel has healed), the plasminogen is activated to form plasmin. Plasmin digests the fibrinogen and fibrin. Prothrombin is converted to thrombin, which in turn catalyzes the conversion of fibrinogen to fibrin so a clot can form.

The nurse is assessing urinary output 24 hours after a prostatectomy for a client with continuous bladder irrigation. What color of output should the nurse expect to find in the drainage bag? A. Red wine colored B. Tea colored C. Amber D. Light pink

Ans: D Rationale: The urine drainage following prostatectomy usually begins as a reddish pink, then clears to a light-pink color 24 hours after surgery.

A nurse in the rehabilitation unit is caring for an older adult client who is in cardiac rehabilitation following an MI. The nurse's plan of care calls for the client to walk for 10 minutes 3 times a day. The client questions the relationship between walking and heart function. How should the nurse best reply? A. "The arteries in your legs constrict when you walk and allow the blood to move faster and with more pressure on the tissue." B. "Walking increases your heart rate and blood pressure. Therefore, your heart is under less stress." C. "Walking helps your heart adjust to your new arteries and helps build your self-esteem." D. "When you walk, the muscles in your legs contract and pump the blood in your veins back toward your heart, which allows more blood to return to your heart."

Ans: D Rationale: Veins, unlike arteries, are equipped with valves that allow blood to move against the force of gravity. The legs have one-way bicuspid valves that prevent blood from seeping backward as it moves forward by the muscles in our legs pressing on the veins as we walk and increasing venous return. Leg arteries do constrict when walking, which allows the blood to move faster and with more pressure on the tissue, but the greater concern is increasing the flow of venous blood to the heart. Walking increases, not decreases, the heart's pumping ability, which increases heart rate and blood pressure and the heart's ability to manage stress. Walking does help the heart adjust to new arteries and may enhance self-esteem, but the client had an MI—there are no "new arteries."

The nurse is planning the care of a client with a nutritional deficit and a diagnosis of megaloblastic anemia. The nurse should recognize that this client's health problem is due to which issue with the red blood cells (RBCs)? A. Production of inadequate quantities of RBCs B. Premature release of immature RBCs C. Injury to the RBCs in circulation D. Abnormalities in the structure and function of RBCs

Ans: D Rationale: Vitamin B12 and folate deficiencies are characterized by the production of abnormally large erythrocytes called megaloblasts. Because these cells are abnormal, many are sequestered (trapped) while still in the bone marrow, and their rate of release is decreased. This results in megaloblastic anemia. This pathologic process does not involve inadequate production, premature release, or injury to existing RBCs.

A nurse is assessing a new client who is diagnosed with peripheral artery disease. The nurse cannot feel the pulse in the client's left foot. How should the nurse proceed with assessment? A. Have the primary care provider prescribe a computed tomography (CT) scan. B. Apply a tourniquet for 3 to 5 minutes and then reassess. C. Elevate the extremity and attempt to palpate the pulses. D. Use Doppler ultrasound to identify the pulses.

Ans: D Rationale: When pulses cannot be reliably palpated, a hand-held continuous wave Doppler ultrasound device may be used to hear (insonate) the blood flow in vessels. CT is not normally warranted, and the application of a tourniquet poses health risks and will not aid assessment. Elevating the extremity would make palpation more difficult.


Conjuntos de estudio relacionados

Cumulative Exam Geometry TEST 90%

View Set

Chapter 10: Infection, Infectious Diseases, and Epidemiology

View Set

Chapter 12: Inventory Management

View Set

CIT: 310 Systems Architecture I @ WKU - On-Demand Study Tool

View Set

Network+ Guide to Networks 7th Edition Chapter 7

View Set

energy, flow and nutrient cycling

View Set